<<

1 General Medicine - Surgery IV year

1. Overal mortality rate in case of acute ESR – 24 mm/hr. Temperature 37,4˚C. Make is: the diagnosis? A. 10-20%; A. Appendicular colic; B. 5-10%; B. Appendicular hydrops; C. 0,2-0,8%; C. Appendicular infiltration; D. 1-5%; D. Appendicular ; E. 25%. E. .

2. Name the destructive form of appendicitis. 7. A 34-year-old female patient suffered from A. Appendicular colic; week ago; no other B. Superficial; gastrointestinal problems were noted. On C. hydrops; clinical examination, a mass of about 6 cm D. Phlegmonous; was palpable in the right lower quadrant, E. Catarrhal appendicitis. appeared hard, not reducible and fixed to the parietal muscle. CBC: leucocyts – 3. Koher sign is: 7,5*109/l, ESR – 24 mm/hr. Temperature A. Migration of the pain from the 37,4˚C. Triple therapy with epigastrium to the right lower cefotaxime, amikacin and tinidazole was quadrant; very effective. After 10 days no mass in B. Pain in the right lower quadrant; abdominal cavity was palpated. What time C. One time ; term is optimal to perform appendectomy? D. Pain in the right upper quadrant; A. 1 week; E. Pain in the epigastrium. B. 2 weeks; C. 3 month; 4. In cases of appendicular infiltration is D. 1 year; indicated: E. 2 years. A. Laparoscopic appendectomy; B. Concervative treatment; 8. What instrumental method of examination C. Open appendectomy; is the most efficient in case of portal D. Draining; pyelophlebitis? E. Laparotomy. A. Plain abdominal film; B. Barium meal; 5. In cases of appendicular abscess is C. US; indicated: D. Termography; A. Laparoscopic appendectomy; E. Doppler ultrasound. B. Concervative treatment; C. Open appendectomy; 9. The complications of acute appendicitis D. Draining, if possible - appendectomy; are all, except: E. Ileo-cecal resection. A. Appendicular infiltration; B. Appendicular abscess; 6. A 34-year-old female patient suffered from C. Enzyme peritonitis; abdominal pain week ago; no other D. Pyelophlebitis; gastrointestinal problems were noted. On E. . clinical examination, a mass of about 6 cm was palpable in the right lower quadrant, 10.A 63-year-old male patient in reasonably appeared hard, not reducible and fixed to the good health suddenly suffered from parietal muscle. CBC: leucocyts – 7,5*109/l, (>38°C) and a painful right iliac fossa tumefaction; no other gastrointestinal 2 problems were noted. On clinical C. Posthospital recovery can be shorter examination, a mass of about 5 cm was in uncomplicated appendicitis. palpable in correspondence of right iliac D. Return to full feeding is less than fossa and appeared hard, not reducible and with open technique. fixed to the parietal muscle. Laboratory E. Wound rate is greater data showed leucocytosis, shift to the left with open technique. and elevated erythrocyte sedimentation rate (74 mm/hr) as pathological findings. 12. A79-year-old man has had abdominal pain Abdominal ultrasound examination for 4 days. An operation is performed, evidenced a fluid in the right lower and a gangrenous appendix is removed. quadrant, with heterogenic echotexture and The stump is inverted. Why does acute a thickening of the ileocecal tract. appendicitis in elderly patients and in Abdominal CT, confirmed the presence of children have a worse prognosis? a complex, predominantly cystic, mass of A. The appendix is retrocecal. large size (6×8 cm) with heterogeneous, B. The appendix is in the preileal mainly peripheral enhancement, the position. adjacent cecum had its wall thickened and C. The appendix is in the pelvic position. it was not possible to differentiate the D. The omentum and peritoneal cavity appendix separately from the mass, appear to be less efficient in homolateral inguinal reactive localizing the disease in these age lymphadenopathy was also present. The groups. patient failed to respond to the initial E. The appendix is longer in these age conservative management, which consisted groups. of intravenous fluids and triple antibiotic therapy with cefotaxime, gentamicin and 13. A 12-year-old boy complains of pain in , without any improvement the lower (mainly on the right of pain and fever. At a further ultrasound side). Symptoms commenced 12 hours examination, the mass appeared not before admission. He had noted modified. Make the diagnosis? during this period. Examination revealed A. Appendicular infiltration; tenderness in the right iliac fossa, which B. Appendicular abscess; was maximal 1 cm below McBurney’s C. Peritonitis; point. In appendicitis, where does the D. Appendicular colic; pain frequently commence? E. Phlegmonous appendicitis; A. In the umbilical region and then moves to the right iliac fossa 11. A 17-year-old female model presents to B. In the back and moves to the right the emergency room with a 1-day history iliac fossa of lower abdominal pain. On examination C. In the rectal region and moves to the she is most tender in the right lower right iliac fossa quadrant (RLQ) and also has pelvic D. In the right iliac fossa and remains tenderness. White cell (WBC) there count is 13x109/l and temperature is E. In the right flank 38,2°C. A provisional diagnosis of uncomplicated appendicitis is made and 14. On examination, patients presenting with laparoscopic appendectomy is offered. appendicitis typically show maximal Regarding laparoscopic appendectomy tenderness over which of the following? which of the following is TRUE? A. Inguinal region A. It can be performed safely with B. Immediately above the umbilicus minimal morbidity compared to C. At a point between the outer one-third and open technique. inner two-thirds of a line between the B. Length of hospital stay is longer than umbilicus and the anterior superior iliac with open technique. spine 3 D. At a point between the outer two-thirds D. Evaluate the upper abdomen for and inner one-third of a line between the or perforated duodenal umbilicus and the anterior superior iliac ulcer spine E. Evaluate for Meckel’s E. At the midpoint of a line between the umbilicus and the anterior superior iliac spine 18. M., 68-year old man, 14 hours ago appeared continuous pain in the RLQ, 2 15. A 29-year-old woman presents to her hours ago pain decreased significantly. physician’s office with pain in the right The diagnosis of acute appendicitis was iliac fossa. Examination reveals made. What morphological form acute tenderness in this region. Her last appendicitis we must suspect? menstrual cycle was 2 weeks previously. A. Gangrenous; CBC: leukocyte count – 7,2x109/l, RBC B. Superficial; – 2,9x1012/l, Hb – 105 g/l. Make the C. Appendix hydrops; diagnosis. D. Phlegmonous; A. Acute superficial appendicitis E. Appendicular colic. B. Ovarian apoplexy C. 19. A 20-year-old man has undergone D. Acute destructive appendicitis appendectomy for perforated E. Acute appendicitis with generalized peritonitis. Seven days postoperatively, his 16. A 28-year-old man is admitted to the temperature continues to spike to 39,5°C emergency department complaining of despite antibiotic therapy with pain in the umbilical region that moves ampicillin, gentamicin, and to the right iliac fossa. Which is a metronidazole. Abdominal CT scan corroborative sign of acute appendicitis? reveals a large pelvic abscess. Soon A. in the right side with afterward, he has from the pressure on the left (Rovsing)sign mouth and nose with increasing oozing B. Increase of pain with testiculalr from the surgical wound and all elevation intravenous puncture sites. What is the C. Relief of pain in lower abdomen with most likely diagnosis? extension of thigh A. Anaphylactoid reaction to intravenous D. Relief of pain in lower abdomen with dye; internal rotation of right thigh B. Antibiotic-induced coagulopathy E. Hyperesthesia in the right lower C. Disseminated intravascular coagulation abdomen D. failure ; E. Congenital bleeding disorder. 17. At open operation a normal appendix is found, no other pathology in abdominal 20. A 20-year-old man has undergone cavity. What is the most common appendectomy for perforated procedure a surgeon should do if he appendicitis with generalized peritonitis. finds a normal appendix? Seven days postoperatively, his A. Evaluate the pelvis for tuboovarian temperature continues to spike to 39,5°C abscess pelvic inflammatory disease, despite antibiotic therapy with malignancy or etopic ampicillin, gentamicin, and B. Removal of appendix metronidazole. Abdominal CT scan C. Evaluate the terminal and reveals a large pelvic abscess. Soon cecum for signs of regional or afterward, he has bleeding from the bacterial mouth and nose with increasing oozing from the surgical wound and all intravenous puncture sites. What was a 4 trigger for coagulopathy? B. Appendicitis occurs rarely in very A. Sepsis; young children and elderly persons. B. Antibioticotherapy; C. Luminal obstruction leads to secretion C. Congenital disoder; of mucus and fluid with the rise in D. Proteolysis; luminal pressure causing of E. Operation. the wall of appendix D. In general, patients with appendicitis 21. All listed below locations of appendix report and loss of appetite concerning cecum are correct, EXCEPT: E. Migration of the pain from epigastric or A. medial paraumbilical region to right lower B. retrocecal quadrant is a sign of patient’s recovery C. subhepatic D. pelvic 26. Typically, in the case of acute E. intermediate appendicitis, symptoms of the disease include: 1) rebound tenderness 22. Appendicular artery is the branch of: (Blumberg’s sign) in the RLQ, 2) A. a. ileocolica hematuria, 3) pain in the RLQ, 4) B. a. mesenterica inferior involuntary guarding, 5) normal body C. a. hepatica communis temperature. Choose the RIGHT D. celiac trunck combination. E. a. iliaca interna A. 1, 3, 4 B. 1, 2, 4 23. What is the Clado ligament? C. 1, 2, 5 A. lig. appendico-ovaricum D. 2, 3, 5 B. lig. appendico-cecalis E. 3, 4, 5 C. lig. appendico-transversum D. lig. circularis appendicis 27. Which sign reflects pelvic location of the E. lig. longitudinalis appendicis appendix? A. Rovsing’s sign 24. Classification proposed by V.I. Kolesov B. Blumberg’s sign in 1972, is based on clinical course of C. disease and includes FOUR items. They D. are all, EXCEPT: E. Koher’s sign A. Appendicular colic B. Simple appendicitis: superficial, 28. You will perform all listed laboratory catarrhal tests to make the diagnosis and C. Destructive appendicitis: of acute phlegmonous, gangrenous, appendicitis, EXCEPT? perforating A. white blood cell (WBC) count D. Complications of acute B. urinanalysis appendicitis: appendicular C. serum liver enzymes and infiltrate, appendicular abscess, levels diffuse peritonitis D. serum HCG (human chorionic E. Purulent complications: , gonadotropin) level in women of sepsis childbearing age E. serum protein 25. Which statement, concerning acute appendicitis, is NOT TRUE? 29. Which statement, concerning acute A. The tip of the appendix can be located appendicitis, is WRONG? anywhere in the right lower quadrant A. Perforation of the appendix is of the abdomen or pelvis accompanied with a sharp abdominal pain 5 B. Acute appendicitis with pelvic D. An elevated amylase level excludes location of the appendix is always this diagnosis characterized by clear clinical E. A contracted is noted on picture and typical course ultrasound C. Often the patients, with acute appendicitis with pelvic location of 33. A 38-year-old male lawyer develops the appendix, have and/or abdominal pain after having a fatty meal. dysuria Examination reveals tenderness in the right D. White blood cells (WBC) are hypochondrium and a positive Murphy’s usually elevated in patients with sign. Which test is most likely to reveal acute appendicitis acute cholecystitis? E. Infiltrate is being formed on 3-5 A. US of the abdomen days of the disease B. Oral cholecystogram C. Intravenous cholangiogram 30. On the 10th day after the admission to the D. Abdominal x-ray surgical department patient develops E. ERCP suppuration of appendicular infiltrate. In this case, clinical picture will include all 34. A 48-year-old woman is admitted to the symptoms, EXCEPT: hospital with severe abdominal pain, A. pain in the region of abdominal tenderness in the right hypochondrium, mass and a WBC count of 12x109/l. Acute B. fatigue cholecystitis is established. After C. headache diagnosis, should be D. hectic fever performed within which of the following? E. breathlessness A. 20–60 minutes 31. In attempting to minimize complications B. The first 1-2 days following hospital during cholecystectomy, the surgeon admission defines the triangle of Calot during the C. 8 days operation. The boundaries of the triangle D. 3 weeks of Calot (modified) are the common E. 3 months hepatic duct medially, the inferiorly, and the liver superiorly. Which 35. A 60-year-old diabetic man is admitted to structure courses through this triangle ? the hospital with a diagnosis of acute A. Left hepatic artery cholecystitis. The WBC count is 28 B. Right renal vein x109/l, and a plain film of the abdomen C. Right hepatic artery and CT scan show evidence of intramural D. Cystic artery gas in the gallbladder. What is the most E. Superior mesenteric vein likely diagnosis? A. 32. A 65-year-old woman is admitted with B. Acalculous cholecystitis RUQ pain radiating to the right shoulder, C. Cholangiohepatitis accompanied by nausea and vomiting. D. Sclerosing cholangitis Examination reveals tenderness in the E. Emphysematous gallbladder RUQ and a positive Murphy’s sign. A diagnosis of acute (uncomplicated) 36. A 60-year-old woman is recovering from cholecystitis is made. What is the most a major pelvic cancer operation and likely finding? develops severe abdominal pain and A. Serum levels may be elevated sepsis. Acute cholecystitis is established, B. Cholelithiasis is present in 90% of all laparotomy is performed. The gallbladder cases is severely inflamed and removed. There C. Bacteria are rarely found at operation is no evidence of gallbladder stones (acalculous cholecystitis). 6 Cholecystectomy is performed. Which is A. Fundus true of acalculous cholecystitis? B. Body A. It is usually associated with stones in C. Infundibulum the CBD. D. Neck B. It occurs in 80–90% of cases of E. Cardia cholecystitis. C. It has a more favorable prognosis than 40. Sometimes small accessory duct may calculous cholecystitis. drain directly into the gallbladder. The D. It is increased in frequency after name of this duct is? trauma or operation. A. Cholecystohepatic duct of Luschka E. It is characterized on US scan by B. Cholecystohepatic duct of inlarged cystic duct. Laschtuvka C. Cholecystohepatic duct of Linka 37. Following recovery in the hospital from a D. Cholecystohepatic duct of Calot fracture of the femur, a 45-year-old E. Cholecystohepatic duct of Koher female patient develops RUQ abdominal pain and fever. She has tenderness in the 41. Different hormones regulate function of right subcostal region. There is evidence the gallbladder. Choose all of them: 1). of progressive sepsis and hemodynamic Vasoactive intestinal polypeptide; 2). instability. The WBC count is 24 x109/l. Cholecystokinin; 3). Thyroxine; 4). A bedside sonogram confirms the Lipase; 5). Motilin. presence of acalculous cholecystitis. A. 1, 2, 3 What should treatment involve? B. 1, 2, 5 A. Intravenous alone C. 1, 3, 4 B. ERCP D. 3, 4, 5 C. Percutaneous drainage of the E. 2, 3, 4 gallbladder D. Urgent cholecystectomy 42. What is the most common symptom E. Elective cholecystectomy after 3 related to acute cholecystitis? months A. RUQ abdominal pain B. 38. Following recovery in the hospital from a C. Fever fracture of the femur, a 75-year-old D. Nausea and vomiting nursing home female patient develops E. RUQ abdominal pain and fever. She has tenderness in the right subcostal region. 43. Which condition does NOT predispose to There is evidence of progressive sepsis gallstone formation? and hemodynamic instability. The WBC A. Concentrated bile count is 24 x109/l. A bedside sonogram B. Increased gallbladder motility confirms the presence of acalculous C. Female sex cholecystitis. What should treatment D. involve? E. Previous truncal vagotomy A. Intravenous antibiotics alone B. Percutaneous drainage of the 44. Patient, who suffers from acute gallbladder cholecystitis, can have all listed below C. ERCP symptoms, EXCEPT: D. Urgent cholecystectomy A. Nausea and vomiting E. Elective cholecystectomy after 3 B. Anorexia months C. Fever D. Arterial hypotention 39. The gallbladder consists of following E. RUQ pain parts, EXCEPT: 7 45. in patient with acute 50. Normal diameter of cystic duct is: cholecystitis, can reveale all signs, EXCEPT: A. 2 mm A. Guarding and rebound tenderness B. 3-4 mm in the RUQ C. 10-12 mm B. Positive Murphy’s sign D. 20 mm C. Painful mass in the RUQ E. 8-9 mm D. Positive Blumberg’s in the RUQ E. Absence of liver dullness 51. Choose the reasons which are responsible for formation during the 46. Which statement concerning imaging pregnancy: studies in patients with acute cholecystitis is A. Hyperestrogenemia, WRONG? B. Decraesed duodenal motility A. The most important value of plain C. Increased gallbladder motility abdominal film is exclusion of other D. Decreased liver function abdominal pathology (perforated E. Hyperglycemia peptic ulcer) B. Ultrasound of the abdomen is an 52. What type of bile stones didn’t exist? effective method for identifying A. Cholesterol gallstones in case of acute B. Pigment cholecystitis C. Mixed C. Gallstones and bile appear nearly D. Calcium isodense on CT E. Protein D. Cholangiography is the most accurate method the extrahepatic biliary tree 53. At : patient is visualization asked to breathe out and then physician gently E. All patients with acute cholecystitis place the hand below the costal margin on the should undergo MRI right side at the mid-clavicular line, the patient is then instructed to breathe in. If the 47. The most efficient noninvasive patient stops breathing in the test is instrumental method of examination in case considered positive. Name this test? of acute cholecystitis is: A. Murphy’s sign A. Oral cholecystography B. Kehr’s sign B. Ultrasonography C. Blumberg’s sign C. Termography D. Rovsing’s sign D. GIT X-ray examination E. Psoas sign E. IV cholangiography 54. Patient: 44 years female. Three weeks ago 48. Which signs are positive in patients with felt acute pain in right upper quadrant acute cholecystitis? after fatty food, spasmolitics were A. Ortner, Kehr, Murphy efficient. Complaints on admission: dull B. Kocher, Sitkovski, Rovzing pain in right upper quadrant. Examination: C. Pasternatski temperature 36,8ºC, soft mass palpated in D. Mayo-Robson Kehr point. CBC: WBC – 6,8x109/l. US: E. Cullen, Grey Turner very large gallbladder with thin wall, stone 15 mm in diameter obstructing it, 49. Normal diameter of CBD is: common 7 mm in diameter. A. up to 25 mm Make the diagnosis? B. up to 15 mm F. Acute cholecystitis; C. up to 10 mm G. Biliary pancreatitis; D. up to 20 mm H. Gallbladder hydropsia; E. up to 5 mm I. Cholangitis; J. Mechanical jaundice. 8 cholecystectomy on a 30-year-old 55. The management in case of acute woman. She has no previous surgical cholecystitis complicated with history. There is a 0.8-cm filling defect in choledocholithiasis should be? the distal (CBD). The A. Conventional cholecystectomy; surgeon should: B. Laparoscopic cholecystectomy; A. Complete the laparoscopic C. ; cholecystectomy and check liver D. ECRP, sphincterotomy, function tests (LFTs) postoperatively. cholecystectomy; If they are normal, no further E. Concervative treatment. treatment is needed B. Complete the laparoscopic 56. To what complication can lead acute obstructive cholecystectomy and repeat an cholecystitis? ultrasound postoperatively. Observe A. Cancer of pancreatic gland; the patient if no CBD stone is B. Pleural empiema; visualized C. Gallbladder empiema; C. Perform a CBD exploration either D. Perforative duodenal ulcer; laparoscopically or open along with a E. Acute bowle obstruction. cholecystectomy D. Complete the laparoscopic 57. Which complication of acute cholecystitis cholecystectomy, no further treatment develops, when the patient have jaundice, is necessary high temperature (>38ºC), pain in upper E. Complete the laparoscopic right quadrant, swetting: cholecystectomy and plan for a A. ; postoperative hydroxy iminodiacetic B. Bile duct ; acid (HIDA) scan C. Gallbladder empiema; 60. A 42-year-old accountant presents with Duodenal ulcer; D. recurrent RUQ pain of 3-year duration. He E. Cholangitis. had undergone a laparoscopic cholecystectomy 2-years ago for 58. An 85-year-old man is brought to the presumed symptomatic cholelithiasis, but hospital with a 2-day history of nausea the pain persisted. An upper GI endoscopy and vomiting. He has not passed gas or is normal. A sonogram and CT scan of the moved his bowels for the last 5 days. abdomen are normal. An ERCP is Abdominal films show dilated small performed, and the pressure in the CBD is bowel, no air in the and air in the 45-cm saline (normal bile duct pressure is biliary tree. Which of the following 10–18- cm saline). What is the most likely statements is TRUE? diagnosis? A. Air in the biliary tree associated with A. Acalculous cholecystitis small- suggests a B. Emphysematous cholecystitis diagnosis of ; C. Biliary B. An enterotomy should be distal to the D. Cancer of the gallbladder site of obstruction and the stone E. Myasthenia gravis should be removed; C Gallstone ileus is more common in the 61. A 42-year-old accountant presents with young adults; recurrent RUQ pain of 3-year duration. He D. Cholecystectomy is contraindicated; had undergone a laparoscopic E. Small-bowel obstruction usually occurs cholecystectomy 2-years ago for presumed in the distal . symptomatic cholelithiasis, but the pain persisted. An upper GI endoscopy is normal. 59. An intraoperative cholangiogram is A sonogram and CT scan of the abdomen are performed during an elective laparoscopic normal. An ERCP is performed, and the 9 pressure in the CBD is 45-cm saline (normal What laboratory finding that supports bile duct pressure is 10–18- cm saline). diagnosis ? Choose the best treatment option: A. Amylase elevation with normal A. Calcium channel blockers, if not findings on liver studies effective an endoscopic B. elevation with sphincterotomy normal or elevated normal bilirubin B. Antibiotics, if not effective an levels endoscopic sphincterotomy C. Elevated serum glutamic C. Analgesics, if not effective an oxaloacetictransaminase (SGOT) endoscopic sphincterotomy levels D. β-blockers, if not effective an D. Altered urea/creatinine ratio endoscopic sphincterotomy E. Urobilin in urine E. No treatment 65. An 70-year-old male presents with a 62. A 43-year-old woman undergoes open clinical diagnosis of acute cholangitis. cholecystectomy. Intraoperative Which organism is most likely involved cholangiogram revealed multiple stones in in the pathogenesis of ascending the CBD. Exploration of the CBD was cholangitis? performed to extract gallstones. The CBD A. Clonorchis sinensis was drained with a #18 T-tube. After 10 B. days, a T-tube cholangiogram reveals a C. Salmonella retained CBD stone. This should be D. treated by which of the following? E. Clostridia A. Laparotomy and CBD exploration B. Subcutaneous heparinization 66. Following admission to the hospital for C. Antibiotic therapy for 6 months and intestinal obstruction, a 48-year-old then reevaluation woman states that she previously had D. ERCP, endoscopic sphincterotomy, undergone cholecystectomy and CBD exploration choledochoduodenostomy. The most E. Ultrasound crushing of the CBD stone likely indication for the performance of the choledochoduodenostomy was: 63. A 62-year-old woman who underwent A. Hepatic metastasis were present cholecystectomy and B. Multiple stones were present in the choledochoduodenostomy (CBD gallbladder at the previous operation duodenal anastomosis) 5 years C. Multiple stones were present in the previously is admitted to the hospital CBD at the previous operation with a 3-day history of upper abdominal D. The common hepatic duct had a pain, , fever, and dark urine. Make stricture the diagnosis: E. The was occluded A. Ascending cholangitis B. Cancer of gallblader 67. In attempting to minimize complications C. Acute during cholecystectomy, the surgeon D. Acute biliary pancreatitis defines the triangle of Calot during the E. operation. The boundaries of the triangle of Calot (modified) are the common 64. A 62-year-old woman who underwent hepatic duct medially, the cystic duct cholecystectomy and inferiorly, and the liver superiorly. choledochoduodenostomy (CBD duodenal Which structure courses through this anastomosis) 5 years previously is admitted to triangle? the hospital with a 3-day history of upper A. Left hepatic artery abdominal pain, chills, fever, and dark urine. B. Right renal vein C. Right hepatic artery 10 D. Cystic artery Intravenous. Choose the most complete E. Superior mesenteric vein answer. A. 1, 2, 3 68. A 55-year-old white female undergoes a B. 1, 3, 4 laparoscopic cholecystectomy for C. 1, 4, 5 symptomatic cholelithiasis. The operation D. All are correct went well, and the patient was discharged E. 2, 4, 5 home. One week later, she comes to your office for a routine postoperative follow- 72. Acute cholangitis is a bacterial up. The final pathology report shows an and most commonly is associated with: 1. incidental finding of a gallbladder Gallstones; 2. Acute ; 3. Malignant carcinoma confined to the mucosa. In tumor of CBD; 4. Truncal vagotomy; 5. further advising the patient, you should Stricture of the CBD. Choose the correct inform her that combination. A. She should undergo radiation therapy A. 1, 2, 3 B She should undergo right hepatectomy B. 1, 3, 4 to remove locally infiltrating disease C. 2, 4, 5 C. She should undergo regional D. 1, 3, 5 lymphadenectomy E. 2, 4, 5 D. She requires systemic E. She does not require any further 73. Which factors play important role in the therapy pathogenesis of acute cholangitis? 1. obstruction; 2. Elevated intraluminal 69. Acute cholangitis is characterized by pressure; 3. Decreased bile production by three symptoms known as Charcot’s hepatocytes 4. Decreased serotonin serum triad. They are: 1). Weight loss; 2). level; 5. Infection of the bile. Choose the RUQ abdominal pain; 3). Jaundice; 4). correct combination. Nausea and vomiting; 5). Fever. Choose A. 1, 2, 3 the correct combination. B. 2, 3, 4 A. 1, 2, 3 C. 3, 4, 5 B. 1, 3, 4 D. 1, 2, 5 C. 1, 4, 5 E. 1, 4, 5 D. 2, 3, 5 E. 2, 4, 5 74. Which condition is NOT associated with higher morbidity and mortality rate in patients 70. Reynolds pentad known as symptoms in with acute cholangitis: patients with severe acute cholangitis. A. Hypotension Which symptom do NOT belong to B. Acute renal failure Reynold’s pentad? C. Liver abscess A. Altered mental status D. Liver B. Arterial hypotension E. Age less than 50 years C. Jaundice D. Nausea and vomiting 75. Which symptom together with Charcot's E. Fever triad is associated with acute cholangitis: A. Coffee round vomitus 71. Cholangiography is the most accurate and B. Pruritus sensitive method of biliary tree C. Hypertention visualization. Diagnostic cholangiogram D. Diarrhea can be performed in different ways. Name E. them? 1). Percutaneously; 2). Endoscopically; 3). Intraoperatively; 5). 76. Which of the following DOESN’T increase the risk of cholangitis: 11 A. CBD stones confirmed on US phosphatase; 3. Elevated C-reactive B. Recent cholecystectomy protein; 4. Elevated ESR; 5. C. Recent ERCP A. All are present D. History of cholangitis in the past B. 1, 2, 3 medical history C. 1, 3, 5 E. Recent appendectomy D. 1, 2, 4 E. 2, 4, 5 77. In patients with cholangitis at physical examination you can reveal the following: 1. 82. On X-ray film air in the biliary tree was RUQ tenderness; 2. Fever; 3. Mental status detected, which condition does NOT cause changes; 4. Hypotension; 5. Tachycardia. this? Choose the correct combination. A. Recent papillotomy A. 1, 2, 3 B. Emphysematous cholecystitis B. 2, 3, 4 C. Cholangitis C. 3, 4, 5 D. Cholecystic-enteric fistula D. All are correct E. Gallbladder hydropsia E. 1, 4, 5 83. Which disorder can cause gallbladder 78. Which statement, regarding acute mucocele? 1. Impacted stone in the cholansitis, is NOT true? gallbladder neck or cystic duct; 2. A. Partial obstruction is associated Spontaneously resolved acute cholecystitis; 3. with a higher rate of infection than Tumors of the CBD; 4. Extrinsic compression complete obstruction of cystic duct by lymph nodes; 5. Ascending B. The bile is normally sterile cholangitis. Choose the correct combination. C. In case of cholangitis obstruction A. 1, 2, 3 of the common bile duct occurs B. 2, 3, 5 secondary to ascending infection C. 3, 4, 5 D. The male-to-female ratio is equal D. 1, 2, 4 in cholangitis E. 1, 4, 5 E. The median age at presentation is between 50 and 60 years. 84. Which statement concerning gallbladder hydrops is WRONG? 79. Which tumor does NOT cause A. Acute is absent cholangitis? B. A large, painful gallbladder is found A. Pancreatic cancer on physical examination B. Cholangiocarcinoma C. Laboratory test are normal C. Ampullary cancer D. Ultrasonography of the RUQ shows an D. Porta hepatis tumors impacted stone in the neck of an E. enlarged gallbladder E. Intraoperatively, the aspirate from the 80. Which disease is NOT responsible for the gallbladder is clear fluid (white bile) development of cholangitis? A. Strictures or stenosis of ampulla Vateri 85. The gallbladder hydrops may develop all B. Endoscopic manipulation of the CBD complication, listed below EXCEPT: C. AIDS cholangiopathy A. of the gallbladder D. Ascaris lumbricoides B. Perforation of the gallbladder E. Later C. Gastric outlet obstruction D. Cholecystenteric fistula 81. What changes in laboratory tests can be E. GIT bleeding find in patients with acute cholangitis? 1. Elevated bilirubin; 2. Elevated alkaline 86. Calculous cholecystitis may develop biliodigestive fistula, which can cause 12 gallstone ileus. In this case fistula is a A. 200-500 ml; connection between gallbladder and..? 1. B. 100-200 ml; ; 2. Sigmoid colon; 3. Transversus C. 500-1000 ml; colon; 4. Stomach; 5. Bladder. Choose the D. 700 ml; correct combination. E. 2500 ml A. 1, 2, 3 B. 1, 3, 4 92. What instrumental examination should C. 3, 4, 5 be performed to diagnose necrotizing D. 1, 2, 5 pancreatitis? E. 2, 4, 5 A. US; B. ECRP; 87. What hormone produces β-cells of C. Plain abdominal film; pancreatic gland: D. Blood gases; A. Somatostatin; E. CT. B. Somatotropin; C. Insulin; 93. A 60-year-old alcoholic is admitted to the D. Glucagon; hospital with a diagnosis of acute E. Pancreozymin. pancreatitis. Upon admission, his white blood cell (WBC) count is 21x109/l. His 88. Which ethiological factors causes acute pancreatitis lipase, lactate dehydrogenase and aspartate most oftenly? aminotransferase are elevated, blood A. Abdominal trauma, alimentary factor; glucose is 10 mmol/L. Which of the B. Gallstones, ; following is TRUE? C. Hypercalcemia, hyperlipidemia; A. This patient is expected to have a D. , toxins; mortality rate of less than 5% E. ERCP, disfunction. B. The patient’s lipase level is not important indication of prognosis 89. Which thesis is correct according to C. This patient requires immediate surgery pathogenesis of acute pancreatitis? D. A venous blood gas would be helpful in A. Intrapancreatic enzyme’s activation; assessing the severity of illness in this B. Pancreatic gland autolisys; patient C. Neutrophils chemoactivation, E. A serum calcium level of 1.7 mmol/L infiltration and inflamation; on the second hospital day is a bad D. SIRS; prognostic sign E. All are correct. 94. A 40-year-old alcoholic male is admitted 90. A 52-year-old male, without alcohol with severe epigastric pain radiating to history, admitted to hospital with acute the back. Serum amylase level is pain in epigastrium, radiating to the reported as normal (fluoroscopic back. Amylase level elevated. Which method), but serum lipase is elevated. radiological findings will help The serum is noted to be milky in diagnosing acute pancreatitis? appearance. A diagnosis of pancreatitis A. Changes in liver on CT scans; is made. The serum amylase is normal B. Choledocholithiasis on US; because: C. Stomach shifted anteriorly on contrast A. The patient has chronic renal failure examination of GIT; B. The patient has hyperlipidemia D. Cloiber cups on plain abdominal film; C. The patient has alcoholic cirrhosis E. Air below diaphragm on plain D. The patient has abdominal film. E. The diagnosis of pancreatitis is incorrect 91. What is the volume of pancreatic juice produced in 24 hours? 13 95. A 57-year-old woman is admitted to the hospital with abdominal pain. She 99. Enzyme toxemia in case of pancronecrosis reports that she drinks alcohol only at is caused by: 1. Trypsin. 2. social occasions. The amylase is Phospholipase A2. 3. Chymotrypsin. 4. elevated to 120 mmol/l. Which Elastase. 5. Enterokinase. Select the following x-ray finding would support a correct combination of answers: diagnosis of acute pancreatitis? A. 1,4 A. Hepatic lesion on CT scan B. 2,3,5 B. Gas in abdominal cavity C. 1,2,3,4 C. Dilated colon transversum D. 1,3,4,5 D. Large loop of colon in the RUQ E. All correct E Irregular cutoff of the CBD on cholangiogram 100. A 45-year-old male addmitted to the surgical department with severe pain in 96. Following a motor vehicle accident a epigastrium with irradiation to the back, truck driver complains of severe which develops after alcohol consumption. abdominal pain. Serum amylase level is Acute pancreatitis was diagnosed. The patient markedly increased to 400 mmol/l. Grey was performed CT scan: pancreatic Turner’s sign is seen in the flanks. enlargement. What is the stage of acute Pancreatic trauma is suspected. Which pancreatitis according to CT picture? statement is true of pancreatic trauma? A. Grade A A. It is oftenly caused by blunt B. Grade B B. It is usually an isolated single-organ C. Grade C D. Grade D C. It often requires a total E. Grade E pancreatectomy D. It may easily diagnosed at operation 101. Acute pancreatitis is most rearly seen in: E. It is proved by the mildly elevated A. > 60 years old women amylase level B. 21-45 years old men C. 45-60 years old women 97. The severity of pancreatitis can be D. 45-60 years old men estimated with: E. Children A. Ranson Criteria B. Alvarado scale 102. How covers pancreatic C. US examination results gland? D. SOFA scale A. From all sides E. CXR B. From anterior and posterior sides C. From anterior and inferior sides 98. Clinical staging (Atlanta consensus) of D. Covers only anterior surface of the gland acute pancreatitis consists of the E. Covers anterior and posterior sides only of following forms of the disease: 1. the head of the gland Pseudotumorous pancreatitis. 2. Mild acute pancreatitis. 3. Sterile 103. For clinical picture of acute pancreatitis pancreonecrosis. 4. Infected are typical all signs except: pancronecrosis. 5. of the A. Continuous severe pain located in the . Select the correct combination epigastrium and/or left upper quadrant of answers: B. The pain irradiates to the back A. 2,3,4 C. Multiple vomiting B. 1,2,3,5 D. Paralytic ileus C. 3,4 E. Absence of liver dullness D. 2,3,4,5 E. All correct 14 104. In differential diagnosis of acute E. Cancer of the head of the pancreatitis and acute bowel obstruction will pancreas. be usefull: 1. CBC; 2. Liver enzymes serum levels; 3. Serum amylase and lipase levels; 4. 109. A 45-year-old male treated in surgical Plain abdominal film; 5. Serum electrolytes. department for acute pancreatitis during Choose best combination. 28 days. He started to complain for the A. 1, 2 temperature elevation to 39°C, B. 2, 3 weakness, fatigue. CBC: RBC – C. 3, 4 3,5*1012/l, WBC – 21*109/l. CT: 11 cm D. 1, 5 in diameter fluid collected near the head E. 2, 5 of the pancreatic gland. What complication of acute pancreatitis 105. Choose changes in biochemical test developed in this case? typical for patients with acute pancreatitis: 1. A. Postnecrotic pseudocyst; Hypertriglyceridemia; 2. Hyperglycemia; 3. B. Retroperitoneal phlegmone; Hypercalcemia; 4. Hypotriglyceridemia; 5. C. Enzyme peritonitis; Hypocalcemia. D. ; A. 1, 2, 5 E. Cancer of the head of the pancreas. B. 1, 3, 4 C. 2, 3, 4 110. The early complications of acute D. 3, 4, 5 pancreatitis are all, except: E. 1, 2, 4 A. Pancreatic ; B. Acute respiratory distress syndrome; 106. Most common cause of death in early C. Enzyme peritonitis; acute pancreatitis is? D. Renal failure; A. Renal failure E. . B. Cardiac failure C. Respiratory failure 111. When infection complications of acute D. Uncontrolled coagulopathy pancreatitis develops most oftenly? E. Sepsis A. On 7-th day of disease; B. After 2 weeks; 107. Severity of acute pancreatitis correlate C. On 4-th day of disease; with levels of all of the following except D. After 6 months; А. Glucose E. On 2-nd day of disease. B. Amylase C. Transaminase 112. Treatment of acute pancreatitis includes D. Calcium all EXCEPT: E. WBC level A. Diurhetics; B. Antibiotics; 108. A 60-year-old female treated in C. Intravenous hydration; surgical department for acute pancreatitis. D. Analgesics; On 30 day of treatment in epigastrium E. Nothing per os. appeared mass 10 cm in diameter. CBC: RBC – 3,7*1012/l, WBC – 10*109/l. US 113. A 24-year-old college student recovers examination: fluid collected near the head from a bout of severe pancreatitis. He of the pancreatic gland. What has mild epigastric discomfort, sensation complication of acute pancreatitis of , and loss of appetite. developed in this case? Examination reveals an epigastric A. Postnecrotic pseudocyst; fullness that on ultrasound is confirmed B. Retroperitoneal phlegmone; to be a pseudocyst. The swelling C. Enzyme peritonitis; increases in size over a 3-week period of D. Pancreatic abscess; 15 observation to 15 cm in diameter. What 117. 30-year-old man hospitalized with a should be the next step in management? severe epigastric pain. During A. Percutaneous drainage of the examination: hypoxemia, dehydration. B. Laparotomy and internal drainage of Laboratory tests: increased levels of the cyst amylase and lipase in the blood. CT C. Excision of pseudocyst confirmed severe acute pancreatitis. D. Total pancreatectomy Which antibiotic will reduce the risk of E. Administration of pancreatic enzymes infection? A. Ampicillin 114. A 42-year-old woman with a history of B. Erythromicin chronic alcoholism is admitted to the C. Gentamicin hospital because of acute pancreatitis. D. Ceftriaxon The bilirubin and amylase levels are in E. Imipenem/cilastatin the normal range. An ultrasound reveals cholelithiasis (stones in the gallbladder, 118. 45-year-old man who abused alcohol, CBD 6 mm in diameter). The symptoms admitted to the hospital complaining of decreased on the fifth day after abdominal pain, nausea, vomiting. Body admission. What should she be advised? temperature 36,8°C. At palpation: A. To start on a low-fat diet at home defined tumor in the epigastrium. B To increase the fat content of her diet Laboratory tests: pancreatic amylase C. To undergo immediate normal. On the second day in the hospital cholecystectomy CT was held: 9 cm pseudocyst of D. To undergo cholecystectomy during pancreas was found. Which statement the same hospital stay as well as an regarding this patient is correct? assessment of her bile ducts А. Pseudocyst can cause compression of E That she will be discharged and should stomach and biliary ducts undergo elective cholecystectomy B. Spontaneous resorption never met after 3 months C. Pseudocyst can be seen only in case of acute pancreatitis 115. A 26-year-old woman with a known D. In this case pseudocyst doesn’t need history of chronic alcoholism is admitted any treatment to the hospital with severe abdominal E. Malignant degeneration occurs in 25% pain due to acute pancreatitis. The serum of cases, if untreated and urinary amylase levels are elevated. On the day following admission to the 119. Which is used to suppress the hospital, there is no improvement, and secretion of the pancreas? she has a mild cough and slight dyspnea. A. Aprotinin What is the most likely complication? B. Octreotide A. Pulmonary atelectasis C. Papaverine B. Bronchitis D. Imipenem/cilastatin C. Pulmonary embolus E. Acetaminophen D. Afferent loop syndrome E. 120. Which drug is used to inhibite proteases? 116. The most often cause of death in case of A. Aprotinin destructive pancreatitis is: B. Octreotide A. Pulmonary embolism C. Papaverine B. Mechanical jaundice D. Imipenem/cilastatin C. Peritonitis E. Acetaminophen D. Septic complications E. Renal failure

16 121. Which statement concerning medical D. Retroperitoneal phlegmon treatment of patient with acute E. Pancreatic abscess pancreatitis on early phase (first 3-5 days) is WRONG? 125. Choose the WRONG statement A. Patients are kept “nothing per os” concerning pseudocyst definition: B. Aggressive fluid resuscitation is A. Peripancreatic fluid collection critically important persisting for more than 4 weeks C. Antibiotics should be used in any case B. Pseudocyst lack an epithelial layer of acute pancreatitis C. Most of them are filled with necrotic D. Imipenem/cilastatin is indicated to debris prevent infication in patients with D. Intervention (minimally invasive or pancreatic conventional) should be performed in E. Early initiation of enteral nutrition is case of pseudocyst complications important in treatment and infection development prevention E. In all cases should be treated with conventional surgery 122. Name the source of infection in patients with necrotizing pancreatitis? 126. Which sign is not associated with peptic A. Pulmonary infection ulcer perforation: B. Bacterial translocation from gut A. Sudden onset; C. Skin infection B. Positive ; D. Urinary infection C. Free air below diaphragm on plain E. Nosocomial infection abdominal film; D. Cloiberg caps; 123. A 44-year-old woman is admitted to the E. Intolerable abdominal pain. hospital because of acute pancreatitis. Biochemical test: bilirubin – 45 mcmol/l. 127. Which ethiological factors causes peptic ulcer An ultrasound reveals stones in disease most oftenly? gallbladder and CBD 14 mm in A. Abdominal trauma, alimentary factor; diameter. MRCP: 8 mm stone in the B. H. pylori, NSAID's; distal part of CBD, which is dilated to 14 C. H. pylori, hyperlipidemia; mm. What should she be the first step in D. Drugs, toxins; the treatment of patient? E. NSAID's, . A. Whipple procedure B Distal resection of the pancreas 128. A 30-year-old male is operated on for C. Sphincterotomy and stone extraction peptic ulcer perforation, in 2,5 hours D. Only medical treatment after the beginning of the disease. E Urgent cholecystectomy Which operation will be most efficient (radical operation for peptic ulcer)? 124. A 47-year-old man with a known history A. Simple closure and highly selective of chronic alcoholism is admitted to the vagotomy; hospital with severe abdominal pain due B. Simple closure with a Graham patch to acute pancreatitis. His general using omentum; condition worsens in first 48 hours. CBC: C. Simple closure; RBC – 4,4*1012/l, WBC – 11,2*109/l. On D. Antrumectomy; second series of CT scans rapid increase E. Stomach resection. in retroperitoneal fluid accumulation was found. Which complication of necrotizing 129. A 42-year-old male, with previous ulcer pancreatitis develops in this case? history and typical clinical picture of A. Enzyme peritonitis peptic ulcer perforation, on examination, B. Retroperitoneal bleeding in 4 hours after the beginning of the C. Pancreatic duct disruption disease, discomfort in right upper 17 quadrant, heart rate – 74/min., mild A. It is performed exclusively via the abdominal wall muscles rigidity, thorax negative Blumberg sign. Free air below B. It can be performed in the neck diaphragm on X-ray abdominal film. C. If complete, it will result in increased What is you diagnosis? acid secretion A. Peptic ulcer recurrence; D. It requires a gastric drainage procedure B. Acute cholecystitis; E. It has been abandoned as a method to C. Covered peptic ulcer perforation; treat ulcer disease. D. Chronic cholecystitis; E. Acute appendicitis (subhepatic 134. A 42-year-old executive has refractory location). chronic duodenal ulcer disease. His physican has suggested several surgical 130. Most oftenly perforated peptic ulcers are options. The patient has chosen a parietal located at: (highly selective) vagotomy instead of a A. Posterior wall of antrum; truncal vagotomy and antrectomy B. Cardiac part of stomach; because? C. Fundus of stomach; A. It results in a lower incidence of ulcer D. Posterior wall of duodenal bulb; recurrence E. Anterior wall of duodenal bulb. B The complication rate is lower C. It reduces acid secretion to a greater 131. Penetrated ulcer can cause such extent complications: 1). Abdominal abscess; D. It benefits patients with antral ulcers the 2). Portal pyelophlebitis; 3). Stomach- most organ fistula; 4). Acute pancreatitis; 5). E. It includes removal of the ulcer Bleeding. A. 1, 2, 3; 135. A63-year-old woman is admitted to the B. 2, 3, 5; hospital with severe abdominal pain of 3- C. 3, 4, 5; hour duration. Abdominal examination D. 1, 3, 5; reveals board-like rigidity, guarding, and E. All. rebound tenderness. Her blood pressure is 90/50 mm Hg, pluse 110 bpm (beats per 132. A 45-year-old man complains of burning minute), and respiratory rate is 30 breaths epigastric pain that wakes him up at per minute. After a thorough history and night. The pain is relieved by eating or physical, and initiation of fluid using over-thecounter antacids and H2 resuscitation, what diagnostic study blockers. Diagnosis is best confirmed by should be performed? which of the following? A. Supine abdominal x-rays A. Urea breath test B. Upright chest x-ray B. Serum gastrin levels C. Gastrograffin swallow C. Barium meal examination D. Computerized axial tomography (CAT) D. Upper endoscopy scan of the abdomen E. Upper endoscopy and biopsy E. Abdominal sonogram

133. A 44-year-old dentist complains of 136. A frail elderly patient is found to have an burning epigastric pain that wakes him up anterior perforation of a duodenal ulcer. at night caused by a recurrent duodenal He has a recent history of nonsteroidal ulcer. He has shown considerable anti-inflammatory drug (NSAID) use and improvement following operative no previous history of peptic ulcer treatment by a truncal vagotomy and disease. A large amount of bilious fluid is pyloroplasty, 10 years prior to this found in the abdomen. What should be incident. Which is TRUE of truncal the next step? vagotomy? 18 A. Lavage and omental patch closure of D. Mallory-Weiss syndrome; the ulcer E. Gastric cancer. B. Lavage of the peritoneal cavity alone C. Total gastrectomy 140. Choose the most efficient diagnostic D. Lavage, vagotomy, and method in case of gastroenterostomy complicated with bleeding: E. Laser of the ulcer A. Abdominal plain film; B. Ultrasound; 137. Name 3 main signs of perforated peptic C. CT scanning; ulcer? D. Endoscopy; A. Abdominal pain, multiple vomiting, E. . distention of abdominal cavity B. Increasing abdominal pain, meteorism, 141. In patient suffering from peptic ulcer peritonitis disease the risk of bleeding is highest in C. Ulcer in past medical history, sudden case of: abdominal pain, board-like rigidity A. Ulcer perforation; D. Ulcer in past medical history, pain in B. Gastric outlet obstruction; the epigastrium, which gradually increases, C. Penetration; peritonitis D. Ulcer malignization; E. Abdominal pain, pale and cold skin, low E. In all cases. BP 142. During endoscopy on the proximal 1/3 of 138. A 63-year-old male patient admitted to lesser curvature of stomach the large hospital with complaints of general ulcer (3x3 cm) with profuse arterial weakness, dizziness, temporary loss of bleeding was visualized. Name the consciousness. For last three years patient bleeding vessel. was complaining of pain in epigastric A. Right gastric artery; area, specially at night, . Medical B. Left gastric artery; examination was not performed. 2 weeks C. Left gastroepiploic artery; before hospitalization, patient complaint D. Right gastroepiploic artery; of intense pain in epigastrium. On the day E. Short gastric arteries. of hospitalization encountered severe weakness, nausea, dizziness, twice black 143. 53-year-old patient, alcoholic, stool with a rotten odor and twice lost of complaining for that consciousness. CBC: hemoglobin – 91 follows episodes of intense vomiting. g/l, RBC – 2,3x109/l. Make the diagnosis? During endoscopy: liner tears below the A. Bleeding from duodenal ulcer; gastroesophageal junction. Make the B. Stomach cancer complicated with diagnosis? bleeding; A. Peptic ulcer complicated with C. Acute pancreatitis; bleeding; D. Myocardial infarction; B. Mallory-Weiss syndrome; E. . C. Esophageal varices; D. Dieulafoy’s lesion; 139. 63-year-old patient admitted with massive E. Gastritis. vomiting with blood, which started suddenly. Previous history of viral 144. Name most often cause of GIT hepatitis. On examination: caput medusae, bleeding? enlarged liver and spleen. What is the A. Peptic ulcer; cause of bleeding? B. Liver cirrhosis; A. Erosive gastroduodenitis; C. ; B. Peptic ulcer disease complicated with bleeding; D. Colon diseases; C. Esophageal varices; E. diseases. 19 E. Surgical intervention, including total 145. The main complication of GIT bleeding gastrectomy is? A. Renal insufficiency; 149. Ahealthy 75-year-old man bleeds from a B. ; duodenal ucler. Medical management C. CNS hypoxia; and endoscopic measures fail to stop the D. Hypovolemic shock; bleeding. What is the next step in E. Centralization of circulation. management? A. Continued transfusion of 8 U of blood 146. What is the most efficient and safe B. Oversewing of the bleeding point, method of bleeding stress ulcers vagotomy. and pyloroplasty treatment? C. Oversewing of the bleeding point A. Stomach resection; D. Administration of norepinephrine B. Sengstaken-Blackmore tube; E. Hepatic artery ligation C. Combination of hemostatic and anti- acid treatment; 150. A73-year-old woman is admitted to the D. Endoscopic coagulation; hospital with a mild UGI hemorrhage E. Gastrotomy and suturing of the that stopped spontaneously. She did not vessels. require transfusion. She had ingested large amounts of aspirin in the past 4 147. Peptic ulcer complicated with bleeding months to relieve the pain caused by is associated with such clinical signs: 1). severe rheumatoid arthritis. Endoscopy Increase of the pain; 2). Coffee ground confirms the presence of a duodenal vomitus; 3). Decrease of the pain; 4). ulcer. A biopsy is done. What is the next Bradycardia; 5). step in the management of a duodenal A. 1, 2, 3; ulcer associated with a positive biopsy B. 2, 3, 5; for H. pylori? C. 3, 4, 5; A. H2 blockers D. 1, 3, 4; B. Bipolar electrocautery of the ulcer E. All. C. Triple therapy D. Photocoagulation 148. A 68-year-old woman has been E. Elective surgery diagnosed with a benign ulcer on the greater curvature of her stomach, 5 cm 151. An 80-year-old grandfather gets proximal to the antrum. After 3 months of admitted to the hospital for a UGI bleed. standard medical therapy, she continues to He undergoes upper endoscopy and have guaiac positive stool, anemia, and bleeding ulcer is visualized. Attempts at abdominal pain with failure of the ulcer to endoscopic cauterization and heal. Biopsies of the gastric ulcer have not epinephrine injection are unsuccessful at identified a malignancy. The next step in stopping the bleeding. A previous management is which of the following? attempt at angioembolization was also A. Treatment of the anemia and repeat all unsuccessful. What is the next definitive studies in 6 weeks step in therapy? B. Endoscopy and bipolar electrocautery A. Elective surgery or laser photocoagulation of the gastric B. PPI IV ulcer C. Blood transfusion C. Admission of the patient for total D. Repeated attempts at bipolar (TPN), treatment of electocanter anemia, and endoscopic therapy E. Emergency surgery D. Surgical intervention, including partial gastric resection 152. A 54-year-old man presents with a massive UGI bleed. After resuscitation, 20 endoscopy is performed. No esophageal C. 3, 4, 5; varices, gastritis, or gastric ulcers are D. 1, 2, 5; seen. After irrigation, a pinpoint lesion is E. 2, 3, 5. seen near the gastro-esophageal junction. Make the diagnosis? 157. The signs of obturation are: 1.Acute A. Mallory-Weiss syndrome onset of disease; 2.Severe abdominal pain; B. Dieulafoy’s lesion 3.Slow beginning; 4.Dull pain; 5. Dilated air- C. Carcinoma filled colon on plain abdominal film. D. Gastro-intestinal stromal tumor A. 1, 2, 3; E. Telangiectasia B. 3, 4, 5; C. 1, 4, 5; 153. A 54-year-old man presents with a D. 2, 3, 5; massive UGI bleed. After resuscitation, E. 1, 2. endoscopy is performed. No esophageal varices, gastritis, or gastric ulcers are 158. Patient with bowel obstruction is seen. After irrigation, a pinpoint lesion is complaining for blood in the stool. What is seen near the gastro-esophageal you diagnosis? junction.What can be said about this A. Intussusception; disease? B. Paralytic ileus; A. It is a carcinoid C. ; B. It is related to alcohol use D. incarceration; C. It is exclusively a mucosal lesion E. Spastic bowel obstruction. D. Surgery if first-line therapy E. Bleeding is from a submucosal vessel 159. For low large bowel obstruction are characteristic all signs except: 154. The most efficient noninvasive A. Slow beginning; diagnostic method for small bowel B. ; obstruction is: C. Cloiberg caps on plain abdominal A. Plain abdominal film; film; B. Laparoscopy; D. Absence of stool; C. Irrigography; E. Acute dehydration. D. Colonoscopy; E. Ultrasound examination. 160. Surgical treatment in case of acute bowel obstruction is indicated in cases: 1. 155. Medical therapy will be effective in cases Electrolyte disbalance. 2. Abdominal cramps. of bowel obstruction caused by: 3. Strangulation. 4. Obstruction has not 1.Volvulus; 2.Cancer of colon; 3.Paralytic resolved within 24-48 hours of concervative ileus; 4.Spastic bowel obstruction; treatment 5. Multiple air-fluid level on plain 5.Coprostasis. abdominal film. A. 1, 2, 3; A. 1, 2; B. 2, 3, 4; B. 3, 4; C. 3, 4, 5; C. 3, 5; D. 1, 2, 5; D. 4, 5; E. 1, 2, 4. E. 2, 3.

156. The signs of strangulation are: 1.Acute 161. A 42-year-old woman is admitted to the onset of disease; 2.Severe abdominal pain; emergency department with severe colicky 3.Slow beginning; 4.Dull pain; 5. Dilated pain, vomiting, and abdominal distention. small intestine loops with air/fluid levels on She has not passed stools or flatus for 48 plain abdominal film. hours. X-rays of the abdomen confirm the A. 4, 5; presence of smallbowel obstruction. What B. 1, 3, 4; 21 is the most likely cause of small-bowel obstruction in the patients? 165. Which sign is characterized by rebound A. Adhesions, inflammatory diseases; tenderness over the site of abnormality in B. Helminths, tumors; patients with peritonitis? C. Groin hernia incarceration, adhesions; A. Kocher’s sign; D. Gallstones, tumors; B. Blumberg's sign; E. Tumors, groin hernia. C. Murphy’s sign; D. Pasternatski’s sign; 162. Most oftenly small bowel obstruction is E. Cullen’s sign. caused by: A. Adhesions, inflammatory diseases; 166. What is the leading etiological factor B. Helmints, tumors; causing spontaneous bacterial peritonitis? C. Groin hernia incarceration, adhesions A. Candida albicans; D. Gallstones, tumors; B. pyogenes; E. Tumors, groin hernia. C. Staphylococcus aureus; D. Escherichia Coli; 163. An elderly nursing home patient is E. . brought to the hospital with recent onset of colicky abdominal pain, distension and 167. Name the scoring system which helps obstipation on examination, the abdomen surgeon to make the prognosis in case of is markedly distended and tympanitic. peritonitis? There is no marked tenderness. Plain A. Manheim index; abdominal x-ray shows a markedly B. APACHE II; distended loop located mainly in the right C. ONTARIO Score; upper quadrant. The likely diagnosis is: D. EuroSCORE; A. Small-bowel obstruction; E. SOFA. B. Chron’s disease; C. Gallstone ileus; 168. A 68-year-old woman is admitted with D. Mesenteric vascular occlusion; an acute surgical abdomen. After E. Sigmoid volvulus. resuscitation with IV crystalloids fluids and administration of 164. 56-year-old patient admitted to the antibiotics, she is taken for an surgical department with complaints of immediate laparotomy. Perforated abdominal pain, repeated vomiting, which of the sigmoid colon is does not bring relief. The pain starts 2 found. The sigmoid colon is inflamed hours before admission, after consumption but mobile and the mesentery contains of large amout of food. Patient anxious, a perforated abscess. The best pale skin, acrocyanosis, pulse 120 bpm, operation for this patient would be: BP 90/60 mmHg. Abdomen moderately A. Insertion of a drain in the abscess; distended in the epigastric region, in the B. Sigmoid resection including the lower parts – sink in. On palpation: abscess and primary colon-to-colon tenderness in the epigastrium. On anastomosis; percussion: tympanic sound in the C. Sigmoid loop colostomy; epigastic region, increased . On D. Left hemicolectomy; plain abdominal film dillated small E. Sigmoid resection and end sigmoid intestinal loops. Make diagnosis? colostomy and oversew the rectum A. Peptic ulcer perforation (Hartmann procedure). B. Groin hernia incarceration C. Acute pancreatitis 169. Which statement is wrong concerning D. Small intestine volvulus primary microbial peritonitis? E. Bowel obstruction caused by the tumor A. Occurs without perforation of a hollow of viscus; 22 B. Occurs with perforation of a hollow E. Positive Ortner sign viscus; C. Caused by direct seeding of 174. Select the method of instrumental microorganisms; examination, which is not suitable for D. Seeding of microorganisms via localization of intra-abdominal abscesses: bacterial translocation from the gut; A. US; E. Seeding of microorganisms via B. Colonoscopy; hematogenous dissemination. C. CT; D. MRI; 170. Which statement is wrong concerning E. Laparoscopy secondary microbial peritonitis? A. Occurs subsequent to perforation of a 175. Specify the main microorganisms, which hollow viscus; are identified in the abdominal cavity of B. Endogenous microbes spill out into the patients with purulent peritonitis: peritoneal cavity; A. Monomicrobial; C. Even after timely surgical intervention B. Gram-positive microorganisms and preemptive antibiotic therapy, domination; about 15-30% of patients demonstrate C. Gram-negative microorganisms ongoing infection consisting of domination; recurrent secondary microbial D. Staphylococcus; peritonitis, intraabdominal abscess, or E. Streptococcus tertiary microbial peritonitis. D. Perforation of the colon is associated 176. Choose a reason for the use of with lower infection rates; metronidazole as a component of E. Patients who develop secondary antibacterial therapy of patients with microbial peritonitis should undergo diffuse peritonitis? surgery to alleviate the source of A. Elimination of anaerobic bacteria; ongoing peritoneal soilage. B. Elimination of gram-positive flora; C. Elimination of gram-negative flora; 171. Which examination would be most D. Elimination of fungal infections; efficient in finding the cause for E. Antiprotozoal antibiotoc secondary bacterial peritonitis? A. Laparoscopy; 177. For the clinical course of acute B. Plain abdominal film; peritonitis three stages are typical. C. Ultrasound; Choose the corect combination: 1. D. CT scan; Subclinical; 2. Reactive; 3. Toxic; 4. E. Level of CRP. Septic; 5. Terminal: A. 1, 2, 3; 172. One of the listed below diseases didn’t B. 2, 3, 4; cause secondary peritonitis C. 1, 2, 5; A. Acute cholecystitis; D. 3, 4, 5; B. Destructive appendicitis; E. 2, 3, 5 C. Acute cholangitis; D. Bowel obstruction; 178. For clinical picture of subdiaphragmatic E. Mesenteric infarction abscess is typical everything except: A. Reduction of pulmonary respiratory 173. Choose the clinical sign, not typical for excursions; acute peritonitis: B. Elevation of diaphragm dome; A. Tachycardia; C. Reactive pleural effusion; B. Positive Blumberg sign; Leukocytosis; C. Muscles rigidity; D. D. Paralytic ileus; E. Hematemesis 23 while in the emergency room. At surgery, a 179. Name the walls of inguinal canal: 1. Richter’s hernia was found. Which of the Inguinal ligament; 2. External oblique following statements is TRUE? abdominal m. aponeurosis; 3. A. It presents lateral to the rectus sheath; Transversalis fascia; 4. Round ligament of B. It presents through the lumbar the uterus; 5. M. rectus abdominis; 6. triangle; Spermatic cord; 7. M. transversus C. It presents through the obturator abdominis, internal oblique abdominal m.. foramen; A. 1, 2, 4, 5; D. It contains a Meckel’s diverticulum; B. 1, 2, 3, 7; E. It may allow normal passage of stool. C. 2, 3, 4, 5; D. 3, 4, 5, 6; 184. At surgery for a right , a E. 4, 5, 6, 7. 72-year-old man is found to have a hernia sac that is not independent of the bowel wall. The 180. A 60-year-old male presents with an cecum forms part of the wall of the sac. Such inguinal hernia of recent onset. Which of the a hernia is properly referred to as which of the following statements are TRUE? following? A. The hernia is more likely to be direct A. Incarcerated; than indirect; B. Irreducible; B. Presents through the posterior wall of C. Sliding; the inguinal canal, lateral to the deep D. Richter’s; inguinal ring; E. Interstitial. C. Is located lateral to epigastric vessels; D. Is more likely than a to 185. Which nerves can be injured during strangulate; surgery for repair of inguinal hernia: 1). E. The sac is congenital. Ilioinguinal n., 2). Pudendal n., 3). Iliohypogastric n., 4). Femoral n., 5). 181. A 70-year-old cigarette smoker presents Popliteal n., 6). Genitofemoral n.. with a right inguinal mass that has enlarged A. 1, 3, 6 and has caused discomfort in recent months. B. 2, 3, 5 The swelling, which does not extend to the C. 1, 2, 3 scrotum, reduces when resting. What is the D. 3, 5, 6 likely diagnosis? E. 1, 5, 6 A. Strangulated indirect inguinal hernia; 186. Which of the following structures would B. Direct inguinal hernia be encountered during repair of an inguinal C. Hydrocele hernia in a male? D. Aneurysm of the femoral artery; A. Spermatic cord; E. Cyst of the cord. B. Round ligament; C. Obturator nerve; 182. Name the best tension-free method for D. Symphysis pubis inguinal hernia repair. E. Nerve to the adductor muscles of the A. Bassini repair; thigh. B. Shouldice repair; C. Stoppa repair; 187. In repair of a femoral hernia, the D. Lichtenstein repair; structure most vulnerable to major E. McVay (Cooper’s ligament) repair. injury lies: A. Medially; 183. A 62-year-old male presents with an B. Laterally; irreducible swelling and severe pain in the left C. Anteriorly; groin. He had a known reducible hernia for 15 D. Posteriorly; years prior to this. He had a bowel movement E. Superficially. 24 large amount of bilious fluid is found in the 188. A 28-year-old professional football abdomen. What should be the next step? player has sudden pain and swelling in A. Lavage of the peritoneal cavity alone the right groin when attempting to B. Lavage and omental patch closure of the intercept a pass. He is admitted to the ulcer local emergency department. On C. Total gastrectomy examination, there is a tender swelling D. Lavage, vagotomy, and in the right groin. The scrotum and gastroenterostomy penis show no abnormality. What is E. Laser of the ulcer the next step in management? A. Needle aspiration to exclude hematoma; 192. A 37-year-old man has had recurrent B. Forceful manual reduction; symptoms suggestive of peptic ulcer disease C. Laparotomy within 20 minutes; for 4 years. Endoscopy reveals an ulcer D. Preoperative preparation and located on duodenum buld. A mucosal biopsy exploration of the groin with hernia reveals Helicobacter. pylori. What is TRUE repair; about H. pylori? E. Morphine and reevaluation within 12 A. Active organisms can be discerned by hours serology. B. It is protective against gastric 189. A 45-year-old man complains of burning carcinoma. epigastric pain that wakes him up at night. C. It is associated with chronic gastritis. The pain is relieved by eating or using over- D. It causes gastric ulcer but not duodenal the-counter antacids and H2 blockers. ulcer. Diagnosis is best confirmed by which of the E. It cannot be detected by the urea breath following? test. A. Urea breath test B. Serum gastrin levels 193. A 68-year-old woman has been C. Barium meal examination diagnosed with a benign ulcer on the greater D. Ultrasound curvature of her stomach, 5 cm proximal to E. Upper endoscopy and biopsy the antrum. After 3 months of standard medical therapy, she continues to have guaiac 190. A 44-year-old patient has refractory positive stool, anemia, and abdominal pain chronic duodenal ulcer disease. His physican with failure of the ulcer to heal. Biopsies of has suggested several surgical options. The the gastric ulcer have not identified a patient has chosen a parietal (highly selective) malignancy. The next step in management is vagotomy instead of a truncal vagotomy and which of the following? antrectomy because? A. Treatment of the anemia and repeat all A. It results in a lower incidence of ulcer studies in 6 weeks recurrence. B. Endoscopy and bipolar electrocautery or B. It benefits patients with antral ulcers the laser photocoagulation of the gastric most. ulcer C. It includes stomach resection. C. Admission of the patient for total D. It left the nerve supply to pylorus intact parenteral nutrition (TPN), treatment (nerves of Latarjet). of anemia, and endoscopic therapy E. It includes removal of the ulcer. D. Surgical intervention, including partial gastric resection 191. A frail elderly patient is found to have an E. Surgical intervention, including total anterior perforation of a duodenal ulcer. He gastrectomy has a recent history of nonsteroidal anti- inflammatory drug (NSAID) use and no 194. Over the past 6 months, a 60-year-old previous history of peptic ulcer disease. A woman with long standing duodenal ulcer disease has been complaining of anorexia, 25 nausea, weight loss and repeated vomiting. B. 1.5 mm Hg She recognizes undigested food in the C. 10 mm Hg vomitus. Examination and workup reveal D. 40 mm Hg dehydration, hypokalemia, and E. 105 mm Hg hypochloremic alkalosis. What is the most likely diagnosis? 198. For how many segments the liver is A. Carcinoma of the fundus divided? B. Penetrating ulcer A. 11 C. Pyloric obstruction due to cicatricial B. 9 stenosis of the lumen of the duodenum C. 8 D. ZES (Zollinger Ellison Syndrome) D. 7 E. Anorexia nervosa E. 5

195. A 2-cm ulcer on the greater curvature of 199. Name the main vessels which forms the stomach is diagnosed in a 70-year-old portal vein woman by a barium study. Gastric analysis to A. Azygos veins maximal acid stimulation shows . B. Hepatic veins What is the next step in management? C. Superior mesenteric vein, A. Antacids, H2 blockers, and repeat inferior mesenteric vein barium study in 6 to 8 weeks D. Splenic vein + inferior B. Proton pump inhibitor (e.g., omeprazole mesenteric vein, superior C. Prostoglandin E (misoprostol) and mesenteric vein repeat barium study in 6 to 8 weeks E. Splenic vein, superior D. Immediate elective surgery mesenteric vein, inferior E. Upper endoscopy with multiple biopsies mesenteric artery (at least 8 or 9) for the ulcer 200. Budd-Chiari syndrome develops due to: 196. A 9-year-old girl had multiple episodes A. Thrombosis of hemorrhoidal of upper GI bleeding. Contrast enhanced CT veins scan showed multiple cavernous B. Thrombosis of hepatic veins malformation surrounding the portal vein. She C. Thrombosis of superior is admitted with severe hematemesis and mesenteric vein melena. At birth, she had developed an D. Thrombosis of inferior infection around the umbilicus. What is the mesenteric vein most likely site of bleeding? E. Thrombosis of splenic vein A. Mallory-Weiss tear of the lower end of the esophagus 201. The portal vein drains blood from the B. Duodenal varices following organs, except: C. Peptic ulcer A. liver D. Esophageal varices B. spleen E. Meckel's diverticulum C. pancreas D. small intestines 197. A 43-year-old man with chronic hepatitis E. stomach and liver cirrhosis is admitted with upper GI bleeding. He has marked and shows 202. What is the most common complication multiple telangiectasias, liver palmar of portal hypertension: erythema, and clubbing. A diagnosis of A. bleeding esophageal varices secondary to B. obstructive jaundice portal hypertension is made. Portal pressure is C. considered elevated when it is above which of D. liver cirrhosis the following? E. variceal hemorrhage A. 0.15 mm Hg 26 203. Determining the cause of portal A. serologies hypertension involves all except (choose one B. Platelet count wrong answer): C. Prothrombin time A. chronic fever D. Albumin B. history of alcohol abuse E. Liver function tests C. umbilical infection D. history of blood transfusions, 208. All patients with cirrhosis should be intravenous drug use (hepatitis considered for the presence of varices at the B and C) time of the initial diagnosis of cirrhosis. E. history of jaundice Gastroesophageal varices confirm the diagnosis of portal hypertension. What 204. Signs of portosystemic collateral examination is the most reliable in revealing formation include the following: 1. the varices: Dilated veins in the anterior abdominal A. antegrade cholangiogram wall (umbilical epigastric vein shunts) 2. B. liver-spleen scan (with Venous pattern on the flanks (portal- technetium sulfur colloid) parietal peritoneal shunting) 3. C. US investigation with Duplex- Paraumbilical hernia 4. Rectal Doppler 5. Ascites - D. upper GI endoscopy and fluid wave 6. Caput medusa (tortuous E. hepatic venous pressure collaterals around the umbilicus).Choose gradient (HVPG) measurement the correct combination of answers: A. 2, 3, 5, 6. 209. Patient with 10-year history of hepatitis B. 1, 2, 3, 5, 6. C is admitted to emergency department with C. 1, 2, 4, 6. massive GI bleeding. Within admission there D. 2, 4, 5, 6. were signs of rebleeding, but patient E. 1, 2, 4, 5, 6. complains on severe weakness and dizziness. What complications can develop in this case : 205. US features suggestive of hepatic 1. , 2. bronchial cirrhosis with portal hypertension include the aspiration, 3. renal failure, 4. portal vein following: 1. nodular liver surface or nodular thrombosis, 5. acute liver necrosis. regenerative hyperplasia. 2. enlarged IVC. 3. A. 1, 3, 4. . 4. "portal" gastropathy. 5. B. 1, 3, 5. ascitis. 6. portal vein cavernous formation. C. 1, 2, 3. Exclude false answers: D. 2, 3, 5. A. 3, 4, 6. E. 2, 4, 5. B. 2, 4. C. 2, 4, 6. 210. 62-year old man with long history of D. 1, 2, 4. liver cirrhosis developed massive variceal E. 1, 3, 5. bleeding. Physician performed upper endoscopy and variceal bleeding was 206. Name scoring system used to evaluate confirmed. First attempt of endoscopic severity of liver cirrhosis: variceal ligation failed. Balloon-tube A. APACHE II tamponade was considered to stop the B. Glasgow score bleeding. For how long Balloon-tube C. Child-Pugh scoring system tamponade is allowed? D. Ranson Criteria A. 6-9 h E. Alvorado score B. up to 24 h C. only within resuscitation 207. To evaluate liver function abnormalities D. 1-2 h all listed below laboratory test should be E. 2-3 days performed, except: 27 211. Choose wrong statement concerning Viral ; 3. Thrombosis of portal blood supply of liver: vein or one of it’s major branches; 4. A. The liver receives a dual blood supply Thrombosis of inferior vena cava 5. Alcohol from both the portal vein and the liver cirrhosis; 6. Umbilical infection in hepatic artery infants. B. 75% of flow from the portal vein and A. 1, 5 25% from the hepatic artery B. 3, 6 C. Venous drainage is via the hepatic C. 1, 4 veins, which drain directly into the D. 3, 5 inferior vena cava E. 4, 6

Portal flow is increased by food intake D. 216. The high mortality associated with first E. The liver receives blood supply only variceal bleeding episodes has led to the from the proper hepatic artery investigation of a variety of methods of prevention of initial bleeding. Which drugs 212. Under the normal conditions per 24 are used to PREVENT initial bleeding by hours the liver produces bile in the volume: lowering the portal preassure? A. 50-100 ml A. α-blockers B. 150-200 ml (phenoxybenzamine, C. 1500-2000 ml phentolamine) D. 600-1000 ml B. β-blockers (propranolol, E. 2000-2500 ml nadolol) C. antibiotics (cefazolin, 213. Choose the reasons which cause ) presinusoidal (prehepatic) portal D. Lactulose hypertension: 1. Budd–Chiari syndrome; 2. E. Diuretics (furosemide) Viral hepatitis C; 3. Thrombosis of portal vein or one of it’s major branches; 4. Thrombosis 217. A 49-year-old man with a history of of inferior vena cava 5. Alcohol liver cirrhosis is admitted with significant cirrhosis; 6. Umbilical infection in infants. hematemesis. There is jaundice and clubbing A. 1, 3 of the fingers. His extremities are cold and B. 3, 6 clammy, and the systolic blood pressure drops C. 1, 4 to 84 mm Hg. After BP stabilization, which D. 3, 5 drugs should be used to STOP bleeding? E. 4, 6 A. phenoxybenzamine or phentolamine 214. Choose the reasons which cause B. propranolol or nadolol sinusoidal (hepatic) portal hypertension: 1. C. furosemide Budd–Chiari syndrome; 2. Viral hepatitis C; D. vasopressin or octreotide 3. Thrombosis of portal vein or one of it’s E. prednisolon or major branches; 4. Thrombosis of inferior hydrocortisone vena cava 5. Alcohol liver cirrhosis; 6. Umbilical infection in infants. 218. A 49-year-old man with a history of A. 1, 3 cirrhosis is admitted with significant B. 3, 6 hematemesis. There is jaundice and clubbing C. 1, 4 of the fingers. His extremities are cold and D. 3, 5 clammy, and the systolic blood pressure drops E. 2, 5 to 84 mm Hg. After BP stabilization, which intervention should be performed primarily to 215. Choose the reasons which cause STOP bleeding? postsinusoidal (posthepatic) portal hypertension: 1. Budd–Chiari syndrome; 2. 28 A. Urgent endoscopy and endoscopic A. Cirrhosis is a histologic bleeding management (band ligation diagnosis, based on three essential or sclerotherapy) criteria: diffuse disease, presence B. Open surgery to stop the bleeding of fibrosis, and replacement of C. Sengstaken-Blakemore tube normal architecture by abnormal D. Emergency lienorenal shunt nodules E. Vagotomy B. Cirrhosis is a reversible process C. Morphologicaly liver cirrhosis can 219. A 42-year-old woman with a known be devided into two groups: history of esophageal varices secondary to macronodular (>3 mm) and hepatitis and cirrhosis is admitted with micronodular (<3 mm) severe hematemesis from esophageal D. Clinicaly cirrhosis is classified to varices. Bleeding persists after two stages: compensated and vasopressin therapy, after endoscopic decompensated bleeding management (band ligation and E. Laboratory tests are performed to sclerotherapy). What should be the next establish the etiology of liver step in management? cirrhosis A. Emergency portocaval shunt B. Emergency lienorenal shunt 222. To determine the etiology of liver C. Splenectomy cirrhosis laboratory test should be performed. D. Vagotomy Which tests will be positive or increased in E. Transjugular intrahepatic case of cholestatic liver cirrhosis? Choose portasystemic shunt (TIPS) correct combination: 1. Alcohol screening; 2. Gamma glutamyl transpeptidase; 3. ALT is 220. A 25-year-old man from rural district on greater than AST; 4. AST is greater than US was diagnosed a cyst 60 mm in diameter, ALT; 5. Viral serology (HBV, HCV); 6. serological test for antigens specific for Alkaline phosphatase. Echinococcus granulosus was positive. Which A. 1, 2 statement is wrong concerning treatment of B. 2, 3 this patient? C. 2, 4 A. Systemic antihelminthic agents are D. 2, 5 generally very effective against E. 2, 6 human Echinococcus B. Surgery remains the standard 223. To determine the etiology of liver approach for hydatid disease of the cirrhosis laboratory test should be performed. liver Which tests will be positive or increased in C. Surgery is indicated in all patients case of alcohol liver cirrhosis? Choose correct with symptomatic disease combination: 1. Alcohol screening; 2. Gamma glutamyl transpeptidase; 3. ALT is greater Surgical treatment should be D. than AST; 4. AST is greater than ALT; 5. considered in patients with Viral serology (HBV, HCV); 6. Alkaline asymptomatic disease discovered phosphatase. accidentally, when the cyst is large A. 1, 2 (>5 cm) B. 1, 3 E. To be curative, surgical therapy C. 1, 4 must remove all the living parasite D. 1, 5 and leave no viable daughter E. 1, 6 or protoscolices 224. To determine the etiology of liver 221. Which statement is wrong concerning cirrhosis laboratory test should be performed. liver cirrhosis? Which tests will be positive or increased in 29 case of viral liver cirrhosis? Choose correct 228. Select the causes of ascites in patients combination: 1. Alcohol screening; 2. Gamma with liver cirrhosis: 1. Portal hypertension glutamyl transpeptidase; 3. ALT is greater 2. AST and ALT high levels 3. than AST; 4. AST is greater than ALT; 5. Secondary hyperaldosteronism 4. Viral serology (HBV, HCV); 6. Alkaline Hypoalbuminemia 5. High level of phosphatase. bilirubin in the blood serum. A. 1, 5 A. 1, 3, 4 B. 2, 5 B. 1, 2, 3 C. 3, 5 C. 2, 3, 5 D. 4, 5 D. 1, 3 E. 5, 6 E. All are correct

225. *In 50-year-old patient., suffering from 229. Which method is the most efficient in Budd-Chiari`s syndrome, appeared diagnosing liver cirrhosis? progressive pain in right subcostal area, A. CT jaundice, varicose veins of the esophagus, B. US rectum, abdominal wall, ascites, C. Radioisotope scanning splenomegaly. Liver cirrhosis was D. Needle biopsy with histological diagnosed. What was the mechanism of examination liver cirrhosis development? E. Biochemical blood test A. Cholestatic hepatitis B. Portal hypertension 230. A patient delivered to the emergency C. department with obstructive jaundice. Choose D. Toxic hepatitis clinical signs which are typical for obstructive E. Viral hepatitis jaundice caused by choledocholisiasis: 1. Upper right quadrant pain; 2. Itch; 3. Back 226. *In patient M. for a long time suffering pain; 4. Vomiting; 5. Discolouration of stool: from liver cirrhosis, recently appeared A. 1, 2, 5 complaints for moderate pain in the B. 2, 3, 5 epigastric area, constant bloating, which C. 1, 4, 5 aggravated after eating. Objective: caput D. 3, 4, 5 medusae on abdominal wall, signs of free E. 1, 2, 3 fluid in the abdominal cavity, enlarged liver and spleen. US: expansion of portal 231. 67-year-old man was made diagnosis cancer of vein, increased liver and spleen. Which pancreas, tumor is located in the head of the gland complication of liver cirrhosis we can and causes obstructive jaundice. Which think about? clinical signs are typical for obstructive A. Portal vein thrombosis jaundice caused by cancer of the pancreas: B. Hepatocellular failure 1. Upper right quadrant pain; 2. Nausea; 3. C. Portal hypertension Palpable enlarged gallblader; 4. Weight D. Peritonitis loss; 5. Discolouration of stool: E. Intestinal dysbacteriosis A. 1, 3, 4 B. 2, 3, 4 227. Choose the symptom of portal C. 1, 2, 3 hypertension: D. 3, 4, 5 A. Tongue papilla atrophy E. 2, 3, 5 B. Jaundice C. Ascites 232. A 58-year-old man with a 20-year D. Erythema of palms history of alcoholism and pancreatitis is E. Increased levels of liver admitted to the hospital with an elevated enzymes in serum bilirubin level to 65 μmol/L, acholic stools, and an amylase level of 120 U*H/L. 30 Obstructive jaundice in case of chronic A. Alkaline phosphatase pancreatitis usually results from which of the B. AST following? C. CRP A. Sclerosing cholangitis D. Protein B. Common bile duct compression E. Amilase caused by inflammation C. Alcoholic hepatitis 237. A 40-year-old man underwent D. laparoscopic cholecystectomy 2 years earlier. E. Splenic vein thrombosis He remains asymptomatic until 1 week before admission, when he complains of RUQ pain 233. A 62-year-old man is admitted with and jaundice. He develops a fever and has abdominal pain and weight loss of 5 kg over several rigor attacks on the day of admission. the past month. He has continued to consume An ultrasound confirms the presence of large amounts of rum. Examination reveals gallstones in the distal CBD. The patient is icteric sclera. The indirect bilirubin level is 50 given antibiotics. Which of the following μmol/L with a total bilirubin of 65 μmol/L. should be undertaken as the next step in An ultrasound shows a 4-cm fluid collection therapy? near the head of the pancreas. What is the A. Should be discharged home under most likely cause of jaundice in a patient with observation alcoholic pancreatitis? B. Should be observed in the hospital A. Alcoholic hepatitis C. Undergo surgical exploration of the B. Carcinoma of pancreas CBD C. Intrahepatic cyst D. ERCP with sphincterotomy and D. stone removal E. Hemolytic anemia E. Anticoagulants

234. A 66-year-old man with obstructive 238. A 49-year-old African American woman jaundice is found on ERCP to have born in New York is admitted with RUQ pain, periampullary carcinoma. He is otherwise in fever, and jaundice (Charcot’s triad.) A excellent physical shape and there is no diagnosis of ascending cholangitis is made. evidence of metastasis. What is the most With regard to the etiology of ascending appropriate treatment? cholangitis, which of the following is TRUE? A. External radiotherapy A. It usually occurs in the absence of B. Local excision and radiotherapy jaundice C. Radical excision (Whipple B. It usually occurs secondary to CBD procedure) when possible stones D. Internal radiation seeds via catheter C. It occurs frequently after E. Stent and chemotherapy choledochoduodenostomy D. It does not occur in patients with 235. Which instrumental examination is cholangiocarcinoma considered to the most efficient and safe in E. It is mainly caused by the liver finding the cause of obstructive jaundice? insufficiency A. CT B. US 239. A surgeon is removing the gallbladder of C. Plain abdominal film a 35-year-old obese man. One week D. MRCP previously the patient had recovered from E. ERCP obstructive jaundice and at operation, numerous small stones are present in the 236. A 67-year-old woman is evaluated for gallbladder. In addition to cholecystectomy, obstructive jaundice. Which biochemical test, the surgeon should also perform which of the except bilirubin level, helps in diagnosing following? obstructive jaundice? A. No further treatment 31 B. Liver biopsy A. 1,6 cm stone in gallbladder; C. Intraoperative cholangiogram B. Cancer of papilla Vatteri; D. Removal of the head of the pancreas C. Residual stone in CBD; E. CBD exploration D. Pancreatic cancer; E. Choledocholithiasis. 240. Choose the diseases for which Courvoisier sign is typical: 1. Chronic calculous 245. Prehepatic jaundice is caused by: cholecystitis; 2. Pancreatic cancer; 3. A. Hemolysis Acute pancreatitis; 4. Cancer of papilla B. Gallstones Vatteri; 5. Liver cirrhosis. C. Pancreatic cancer А. 1, 2, 5; D. Renal fealure В. 3, 5; E. Pancreatic preudocyst С. 2, 5; D. 2, 3, 5; 246. Posthepatic jaundice can be caused by all Е. 2, 4. except: A. Gallstones 241. Name the indications for Intraoperative B. Cholangitis cholangiography: 1. Detection of stones C. Chronic pancreatitis in CBD during palpation; 2. Suspicion D. CBD malignancy for scar narrow of papilla Vatteri; 3. E. Acute hepatitis Jaundice before surgical intervention; 4. Diameter of CBD more than 10 mm; 5. 247. Extrahepatic biliary tree consists of all Jaundice during surgical intervention. except: А. 1, 2, 3, 4; A. Wirsung’s duct В. 1, 3, 4; B. Common hepatic duct С. 3, 4; C. Cystic duct D. 1, 3, 4, 5; D. Gallbladder Е. All are correct. E. Common bile duct

242. Choose the most serious complications of 248. What is the most common benign causes mechanical jaundice: of obstructive jaundice? А. violation of absorption in small A. Gallstones intestine; B. Strictures В. reduction of protein-synthesizing C. Chronic alcohol abuse function of liver; D. Primary sclerosing cholangitis С. hepatic-renal failure; E. Liver cirrhosis D. gallbladder empyema; Е. gallbladder hydrops. 249. What is NOT the additional source of bilirubin production? 243. Choose the indications for choledochotomy: 1. A. Ineffective erythropoiesis Biliary dyskinesia; 2. Acute cholangitis; B. Myoglobinemia 3. Gallbladder perforation and peritonitis; C. Cytochromes metabolism 4. Obstructive jaundice; 5. Detection of D. Large hematoma lysis stones in CBD during palpation. E. Gallstones dissolving А. 1, 3, 4; В. 2, 3, 4; 250. All diseases can cause hemolysis, С. 2, 3, 4, 5; except: D. 2, 4, 5; A. Autoimmune disorders Е. All are correct. B. Hypersplenism C. Defects in hemoglobin structure 244. Which disease did not cause obstructive D. jaundice? E. Iron deficiency anemia 32 A. Cholangitis 251. What are the most common causes of B. Biliary leakage – biliary peritonitis hepatitis? 1. Portal hypertention. 2. Alcohol. C. Pancreatitis 3. Autoimmune disorders. 4. Cholelithiasis. 5. D. Bleeding Viruses. E. Obstructive jaundice A. 2,3,5 B. 1,2,4 257. Which statement is NOT true? C. 3,4,5 A. Intrahepatic disorders can lead to D. 1,3,4 unconjugated-conjugated E. 1,2,5 hyperbilirubinemia B. Posthepatic disorders can cause 252. Chronic alcohol abuse may result in: 1. conjugated hyperbilirubinemia Fatty liver (steatosis). 2. Biliary tract C. Bilirubin is a breakdown product of infection. 3. Gallstone formation. 4. Cirrhosis. heme 5. Hepatitis. D. Pancreatic cancer is associated with A. 2,4,5 positive Courvoisier's sign B. 1,4,5 E. Common bile duct is divided into two C. 2,3,4 parts: supraduodenal and D. 1,2,3 retroduodenal E. 1,3,5 258. All diseases listed below can lead to 253. Classicaly cholangitis is diagnosed extrahepatic biliary obstruction, clinically as syndrome known as Charcot’s EXCEPT: triad. A. Choledocholithiasis A. Fever, hypotension and jaundice B. Acute virual hepatitis B. Fever, RUQ pain and hypotension C. Ampullary cancer C. Pruritus, RUQ pain and jaundice D. Pancreatic cancer D. Pruritus, hypotention, and jaundice E. Biliary strictures E. Fever, RUQ pain, and jaundice 259. Decompression of extrahepatic biliary 254. Which enzymes are markers of biliary obstruction can be achieved by three hypertension? 1. Alkaline phosphatase. 2. methods: 1. Surgical bypass. 2. gamma glutamyl transpeptidase. 3. Percutaneous insertion of stents. 3. Enterocinase, 4. Acid phosphatase. 5. AST. Endoscopic insertion of stents. 4. TIPS A. 2,4 procedure. 5. Nasogastric decompression. B. 1,5 A. 2,4,5 C. 2,3 B. 1,4,5 D. 1,2 C. 2,3,4 E. 3,5 D. 1,2,3 E. 1,3,5 255. Bile is produced by the liver and contains all components, EXCEPT: 260. Which statements, concerning jaundice, A. Bile salts. is NOT true? B. Water and electrolytes. A. High levels of circulating bile salts are C. Cholesterol. associated with pruritus D. Bilirubin. B. Jaundice, dark urine, discoloration of E. Vitamins A, K, D stool and pruritus are the hallmark of obstructive jaundice 256. ERCP and percutaneous transhepatic C. History of fever, and cholangiography are invasive and can be intermittent jaundice may be associated with following complications, suggestive of cholangitis or EXCEPT: choledocholithiasis 33 D. Weight loss, , pain E. 1, 3, 4 radiating to the back and progressively deepening jaundice may be suggestive 266. Which instrumental examination is a of pancreatic cancer reference standard diagnostic method for E. A palpably enlarged gall bladder in a chronic pancreatitis? jaundiced patient is also suggestive of A. US chronic alcohol abuse B. CT C. MRCP 261. Complications of obstructive jaundice D. Plain abdominal film are all of the following, EXCEPT: E. ERCP A. Sepsis caused by cholangitis B. Biliary cirrhosis 267. Choose indications for surgical treatment C. Pancreatitis of CP: 1. Pseudocyst formation; 2. Disabling D. Renal and liver failure abdominal pain; 3. Disabling obstruction of E. Respiratory failure adjacent hollow viscera; 4. Exocrine insufficiency; 5. Diabetus mellitus. 262. Chronic pancreatitis (CP) is A. 1, 2, 5 characterized by (choose correct clues of CP B. 2, 3, 4 pathogenesis): 1. Parenchymal fibrosis; 2. C. 3, 4, 5 Ductal strictures; 3. Atrophy of acinar and D. 1, 2, 3 islet tissue; 4. Pseudocyst formation; 5. CBD E. 1, 2, 4 compression. A. 1, 2, 3 268. Longitudinal pancreaticojejunostomy is B. All correct also known as: C. 2, 3, 4 A. Puestow procedure D. 3, 4, 5 B. Whiple procedure E. 1, 2, 4 C. Billroth reconstruction D. Bassini’s operation 263. Name main forms of CP: 1. Proliferative; E. Hartman’s procedure 2. Calcific; 3. Degenerative; 4. Obstructive; 5. Inflamatory. 269. The most common complication of CP A. 1, 2, 3 is: B. 2, 3, 4 A. Mechanical jaundice C. 2, 4, 5 B. Compression of duodenum D. 3, 4, 5 C. Pseudocyst E. 1, 4, 5 D. Diabetus mellitus E. Exocrine isufficiency 264. The most often reason causing CP is: A. Alcohol ingestion 270. A 40-year-old woman with severe B. Gallstones chronic pancreatitis is scheduled to C. Pancreatic cancer undergo an operation, because other forms D. Autoimmune disorders of treatment have failed. The ultrasound E. Trauma of the pancreas shows no evidence of pseudocyst formation or cholelithiasis and endoscopic 265. Name main clinical signs of CP: 1. retrograde cholangiopancreatogram Epigastric pain; 2. Vomiting; 3. Poor appetite; (ERCP) demonstrates dilated main 4. Weight loss; 5. Positive Blumberg sign in pancreatic duct (12 mm) with multiple RUQ stricture formation. Which operation is A. 1, 2, 3 suitable to treat this condition? B. 2, 3, 4 A. Gastrojejunostomy C. 3, 4, 5 B. Pancreaticojejunostomy D. 2, 4, 5 C. Cholecystectomy 34 D. Splenectomy C. Trypsin and chymotrypsin, which E. Subtotal pancreatectomy function to degrade proteins D. Enterokinase, which functions to 271. A 45-year-old patient with chronic hydrolaze proteins pancreatitis is suffering from malnutrition E. Nucleases such as deoxyribonuclease and weight loss secondary to inadequate and ribonuclease, which function to pancreatic exocrine secretions. Which is break down DNA and RNA, TRUE regarding pancreatic secretions? respectively A. Pancreas releases fluid poor in enzymes 275. Choose TWO main etiological factors B. Pancreas releases fluid rich in causing chronic pancreatitis: 1. enzymes and bicarbonate Alcohol; 2. Autoimmune disorders; 3. C. Cholecystokinin doesn’t have Hyperlipidemia; 4. Pancreatic duct influence into pancreatic secretion obstruction; 5. Idiopathic. D. All pancreatic enzymes are secreted A. 1, 2 in an active form B. 1, 3 E. The pancreas produces proteolytic C. 1, 4 enzymes only D. 1, 5 E. 2, 5 272. Which statement is WRONG concerning anatomy of the pancreas? 276. All problems that compromise blood A. Pancreas is located mesoperitoneally flow listed below can cause acute visceral B. Pancreas consists of head, neck, ischemia, except? body and tail A. Acute embolic occlusion C. The head of the gland lies nestled in B. Acute thrombotic occlusion the C-loop of the second part of the C. Nonocclusive mesenteric ischemia duodenum D. Splanchnic artery aneurysm D. The tail of the gland extends E. Mesenteric veins thrombosis obliquely into the hilum of the spleen 277. Which clinical sings are typical for early E. The neck of the pancreas overlies the stages of acute visceral ischemia? spine, where it is susceptible to A. Severe abdominal pain, vomiting, injury in blunt abdominal trauma diarrhea, leukocytosis B. Mild abdominal pain, constipation 273. The islets of Langerhans are small C. Pulsating abdominal mass islands of endocrine cells within a sea of D. No typical clinical signs exocrine tissue, they does NOT consists E. Bloody stool, signs of peritonitis of: A. A-cells 278. Name later manifestations of acute B. B-cells visceral ischemia? C. C-cells A. Severe abdominal pain, vomiting, D. PP-cells diarrhea, leukocytosis E. D-cells B. Mild abdominal pain, constipation C. Pulsating abdominal mass 274. The pancreas has a major exocrine D. No typical clinical signs function in the production of digestive E. Bloody stool, signs of peritonitis enzymes. These include: A. Amylase, which functions in the 279. Which examination is considered to be a breakdown of starches gold standard for diagnosis of acute visceral B. Lipase, which functions to hydrolyze ischemia? fatty acids A. Ultrasound B. CT 35 C. Selective mesenteric angiography C. Mesenteric adenitis D. Duplex ultrasonography D. Cancer E. MRI E. Pseudomyxoma peritonei

280. Which examination is considered to be 285. Name the form of acute mesenteric ischemia the best for screening of acute visceral which has the highest mortality rate? ischemia caused by thrombotic ischemia or A. Acute embolic occlusion venous thrombosis? B. Acute thrombotic occlusion A. Ultrasound C. Nonocclusive mesenteric ischemia B. CT D. Splanchnic artery aneurysm C. Selective mesenteric angiography E. Mesenteric vein thrombosis D. Duplex ultrasonography E. MRI 286. The intestine is viable in over 90% of patients if the duration of mesenteric ischemia 281. A 60-year-old man with a history of symptoms lasts: atrial fibrillation is found to severe abdominal A. 12 hours or less pain, vomiting, diarrhea, WBC=23*109/l. The B. 24 hours or less embolus is most probably originating from C. 36 hours or less which of the following? D. 48 hours or less A. An atherosclerotic plaque E. 72 hours or less B. An abdominal aortic aneurysm C. Heart 287. Superior mesenteric artery embolism can D. Lungs be caused by: 1. Aneurysm of the left E. Paradoxical embolus ventricle after myocardial infarction; 2. Atrial fibrillation; 3. Bacterial 282. A 60-year-old man with a history of endocarditis; 4. Right ventricle atrial fibrillation is found to severe abdominal hypertrophy; 5. Pulmonary artery pain, vomiting, diarrhea, WBC=23*109/l. stenosis. Choose correct combination: What is the most appropriate surgical А. 1, 3, 4; treatment for this patient? B. 1, 2, 3; A. Embolectomy C. 2, 3, 4; B. Lumbar sympathectomy D. 2, 3, 5; C. Bypass surgery E. All are correct. D. Intestine resection E. Heparinization 288. What can cause infarction of the intestine: 1. Superior mesenteric artery 283. Name the most often cause for embolism; 2. Superior and inferior mesenteric mesenteric thrombosis? artery embolism; 3. Superior mesenteric A. Blunt abdominal trauma artery thrombosis; 4. Superior mesenteric vein B. Arteriosclerotic plaque thrombosis; 5. Prolonged spasm of small C. Mesenteric artery aneurysm intestine arteries. Choose the BEST D. Embolus combination. E. Vasospasm A. 1, 2, 3 B. 2, 3, 4 284. A 66-year-old woman is admitted for C. 3, 4, 5 hyperalimentation due to malnutrition D. All diseases can cause intestine consequent to massive small- resection. What is the most likely E. None of these diseases can condition that leads to the need to cause intestine infarction perform a massive resection? A. Autoimmune disease 289. In a patient with superior mesenteric B. Mesenteric ischemia artery embolism in stage of bowel infarction 36 (part of small intestine necrotised) should be 293. Most often cause of acute mesenteric performed the following operation: ischemia is: A. Thrombectomy A. Embolisation to the superior B. Isolated embolectomy mesenteric artery C. Embolectomy and resection of B. Thrombosis of superior mesenteric necrotised intestine artery D. Total colectomy C. Nonocclusive mesenteric ischemia E. Periarterial sympathectomy D. Portal vein thrombosis E. Mesenteric vein thrombosis 290. A 42-years-old patient, who suffers from mitral stenosis and atrial fibrillation, 6 hours 294. What can cause nonocclusive mesenteric ago appeared severe abdominal pain, ischemia (NOMI)? 1. Severe mesenteric vomiting, diarrhea. On examination: vasoconstriction; 2. Myocardial infarction; 3. tenderness in mesogastrium, negative Septic shock; 4. Hemorrhagic shock; 5. Blumberg sign. CBC: Leukocytes – 21*109/l. Cardiac decompensation. What causes acute mesenteric ischemia in this A. 1, 2, 3 case? B. 2, 3, 5 A. Acute embolic occlusion of C. 2, 3, 4 superior mesenteric artery D. None of these diseases cause B. Acute thrombotic occlusion of NOMI superior mesenteric artery E. All these diseases cause NOMI C. Nonocclusive mesenteric ischemia D. Portal vein thrombosis 295. In patients with acute mesenteric E. Mesenteric vein thrombosis ischemia due to mesenteric embolism, which of the following statements is 291. Which parts of the GI tract will be correct? ischemic in case of thrombosis of the orifice A. Most oftenly embolization to of superior mesenteric artery? inferior mesenteric artery is A. Stomach and duodenum observed B. Stomach, duodenum and ileum B. Embolus most oftenly origins C. Small intestine, cecum, colon from right heart ascendance C. Thrombolytic therapy may be D. Colon and rectum attempted in patients without E. All parts of small and large signs of bowel infarction or intestine gastrointestinal bleeding D. Arteriography usually reveals 292. Which operations we can perform in case the embolus lodged at the of embolic occlusion of superior mesenteric orifice of the superior artery (choose the best combination): 1. mesenteric artery Embolectomy; 2. Embolectomy and resection E. At the time of exploration in of part of small intestine; 3. Embolectomy case of superior mesenteric and left hemicolectomy; 4. Embolectomy and artery embolism, ischemia is right hemicolectomy; 5. Total excision of most severe in the left colon ileum, jejunum and right hemicolectomy. A. 1, 2, 3 296. A 68-year-old man is admitted to the B. 2, 3, 4 coronary care unit with an acute C. 1, 4, 5 myocardial infarction. His postinfarction D. 1, 2, 4 course is marked by congestive heart E. All operations can be failure and intermittent hypotension. On performed the fourth hospital day, he develops severe midabdominal pain. On physical examination, blood pressure is 90/60 37 mm Hg and pulse is 110 beats/min and patient may develop regular; the abdomen is soft with mild mellitus generalized tenderness and distention. E. Fat results in Bowel sounds are hypoactive; stool hematest is positive. The next step in this patient’s management should be which 299. Choose the combination of instrumental of the following? examinations, which can be used in A. Barium enema patients with chronic pancreatitis? 1. B. Upper gastrointestinal Enoscopic US; 2. CT; 3. MRCP; 4. Plain endoscopy abdominal film 5. ERCP. C. Angiography A. 1, 2, 5 D. Ultrasonography B. 1, 3, 5 E. Celiotomy C. 1, 4, 5 D. 2, 4, 5 297. The earliest symptom of CP is E. All can be used abdominal pain, choose WRONG statement concerning the pain in case of 300. In patients with chronic pancreatitis chronic pancreatitis: surgical treatment is indicated in: 1. A. In CP, two pain patterns have been Disabling pain, which interferes with the described: continuous and patient’s ability to work, is refractory to intermittent pancreatic enzyme therapy, requires high B. When pain is intermittent, episodes doses of oral narcotics; 2. Diabetes may be separated by pain-free mellitus; 3. Malabsorption; 4. ERCP intervals of months or years evidence of a dilated pancreatic duct (> C. Episodes of continuous pain last 6 mm); 5. Obstruction of CBD. from daily to 2- to 3-days per week A. 1, 2, 3 for at least 2 months B. 1, 3, 4 D. Pain appeares in the epigastric C. 1, 4, 5 region at the beginning of disease D. 2, 3, 4 with a subsequent shift to the right E. All are indications for surgical iliac region treatment E. Pancreatic pain is felt in the epigastrium or upper abdomen, 301. In diagnosing CP in its early stages, with penetration to the back or clinicians should recognize that: radiation to the left intercostal A. Pain is present, to some degree, in region all patients B. Pain may be absent in 10% and more 298. With advancing disease, patients develop patients exocrine and endocrine insufficiency and C. Direct pancreatic secretory thus lose the ability to digest protein and assessment remains the gold fat. Which statement is WRONG? standard for diagnosing pancreatic A. The patien manifests diabetes insufficiency and is performed mellitus and malabsorption on early frequently (revesible) stage D. Radiographic imaging confirms B. Protein malabsorption results in pancreatitis in early stages of the creatorrhea disease C. Since malnutrition impairs E. Endoscopic US is gold standard immunity, incidence of infection is instrumental examination for likely to rise among affected making the diagnosis chronic patients pancreatitis D. With the destruction of insulin- producing pancreatic cells, the 38 302. Name the most often surgical procedure on the large intestine. complication of chronic Is the diagnosis more likely to be pancreatitis: ulcerative rather than Crohn’s A. Left sided pleural effusion disease because at the previous operation? B. Disseminated intravascular A. The serosa appeared normal on coagulation inspection, but the colon mucosa was C. Splenic vein thrombosis extensively involved D. Pancreatic pseudocyst B. There was evidence of fistula E. Pancreatic ascities formation C. All layers of the bowel wall were 303. All statements concerning Chron’s involved disease are true, except? D. Skip lesions were noted A. Affects all parts of GI (from the E. The preoperative GI series showed a oropharynx to the anus) narrowing string like stricture in the B. Nonspecific inflamation ileum (string sign) C. Superficial (mucosal) inflamation D. Ileocolic region affected most 308. A 35-year-old man has known ulcerative oftenly colitis. Which of the following is an E. Etiology is not known indication for total proctocolectomy? A. Occasional bouts of colic and 304. All statements concerning Ulcerative diarrhea colitis (UC) are true, except? B. Sclerosing cholangitis A. Affects colon and rectum C. Toxic B. Transmural inflamation D. Arthritides C. 20-30% of patients with UC have E. Iron deficiency anemia another family member with the disease D. Etiology is not known 309. A 54-year-old man with diarrhea is E. Smokers are in the risk group for UC found to have . Colectomy should be advised in patients with 305. The typical complications of Chron’s ulcerative colitis who have symptoms that disease are all, except: persist for more than which of the A. Abdominal abscesses following? B. Internal fistula A. 1 month C. Intestinal obstruction caused by B. 6 months strictures C. 1 year D. Sclerosing cholangitis D. 10–20 years E. Hemorrhage E. More than 25 years

306. A 26-year-old man is present with mild 310. A 48-year-old woman develops colon clinical signs of Chron’s colitis. What primary cancer. She is known to have a long history treatment this patient should be of ulcerative colitis. In ulcerative colitis, recommended: which of the following is a characteristic of A. Right hemicolectomy colon cancer? B. Left hemicolectomy A. Occurs more frequently than in the C. Treatment of anemia rest of the population. D. Total coloproctectomy B. Is more likely to occur when the E. Medical treatment with ulcerative disease is confined to the aminosalicylates left colon. C. Occurs equally in the right and left 307. A 43-year-old woman undergoes side. investigation for colitis. In her history, it is D. Has a synchronous carcinoma in 4– noted that 20 years earlier she underwent a 5% of cases. 39 E. Has an excellent prognosis because 315. The typical complications of perirectal of physician awareness. abcess are all, except: A. Internal fistula 311. A 40-year-old man with a long history of B. Hemorrhoids bloody diarrhea presents with increased C. Sphincter injury abdominal pain, vomiting, and fever. On D. Perineal sepsis examination, he is found to be dehydrated E. Chronic fistula and shows tachycardia and hypotension. The abdomen is markedly tender with 316. What imaging study is necessary to guarding and rigidity. What is the most make diagnosis uncomplicated perirectal likely cause? abscess fistula disease? A. in ulcerative colitis A. Sinogram B. Small-bowel perforation from B. Transrectal US regional enteritis C. CT C. Perforated carcinoma of the sigmoid D. No imagine study colon E. MRI D. Volvulus of the sigmoid colon E. Acute perforated diverticulitis 317. The most effective treatment, which is successful in healing 90% of anal fisures, 312. A 25-year-old man has recurrent, includes: 1. Stul softeners; 2. Lexatives; 3. indolent fistula in ano. He also complains Antibiotics; 4. NSAIDs; 5. Sitz bath. of weight loss, recurrent attacks of diarrhea A. 1, 2, 3 with blood mixed in the stool, and B. 1, 2, 4 tenesmus. Proctoscopy revealed a healthy, C. 2, 3, 4 normal-appearing rectum. What is the most D. 3, 4, 5 likely diagnosis? E. 1, 2, 5. A. Colitis associated with acquired immunodeficiency syndrome (AIDS) 318. Uncomplicated interanal hemorrhoids B. Ulcerative colitis typicaly are acossiated with: C. Amoebic colitis A. Anorectal pain D. B. Pain after defecation E. Crohn’s colitis C. Thrombosis D. Perirectal abscesses 313. Perirectal abscess fistulous disease is E. Bright-red bleeding per rectum most oftenly: A. Associated with a specific systemic 319. Interanal and external hemorrhoids can disease develop all complications except: B. Associated with specific infection A. Incarceration disease B. Necrosis C. Cryptoglandular in origin C. Perianal condylomas D. Associated with hemorrhoids D. Thrombosis E. Etiology is not known E. Bleeding

314. According to their location perirectal 320. Protoscopy reveals nonbleeding grade I abscesses are classified to (one answer is hemorrhoids. Indications for surgical not correct): treatment are all except: A. Superficial A. III-IV grade hemorrhoids B. Supralevator B. Severe bleeding C. Ischiorectal C. Thrombosis D. Intersphincteric D. I-II grade hemorrhoids E. Perianal E. Necrosis

40 321. Why during hemorrhoidal bleeding the blood is bright-red? A. Hemorrhoidal vein have lots of shunts with rectal arteries B. Hemorrhoids never bleed C. Hemorrhoids develops from arteries D. Bleeding is associsted with coagulopathy E. Most oftenly upper parts of colon are bleeding, which are richy vascularized

322. A 44-year-old man has recurrent hemorrhoids. What treatment modality is not indicated in case of recurency? A. Conventional surgery B. Minimaly invasive treatment C. Increasing dietary fiber D. decreasing constipating foods, E. decreasing time spent on the toilet